Chapter 35 - Dysrhythmias - Complex 2021

Pataasin ang iyong marka sa homework at exams ngayon gamit ang Quizwiz!

In which part of the cardiac cycle is an ectopic impulse the greatest risk to the patient? 1 3 4 5

5 The patient is at greatest risk if the ectopic impulse falls on the T wave of a preceding beat. This is called the R-on-T phenomenon. This is especially dangerous because the premature ventricular contraction (PVC) is firing during the relative refractory period of ventricular repolarization. Excitability of the heart cells increases during this time, and the risk for the PVC to start ventricular tachycardia (VT) or ventricular fibrillation (VF) is great.

A patient has sought care following a syncopal episode of unknown etiology. Which item will be included in the patient's treatment plan? A head-up tilt-test IV β-adrenergic blocker Pacemaker insertion Antiplatelet therapy

A head-up tilt test. In patients without structural heart disease, the head-up tilt-test is a common component of the diagnostic workup following episodes of syncope. IV β-adrenergic blockers are not indicated, although an IV infusion of low-dose isoproterenol may be started in an attempt to provoke a response if the head-up tilt-test did not produce a response. Addressing pacemakers is premature at this stage. There is no data to support the initiation of antiplatelet therapy.

Which finding from a newly admitted adult patient's electrocardiogram (ECG) requires further investigation by the nurse? a. Isoelectric ST segment b. PR interval of 0.18 second c. QT interval of 0.38 second d. QRS interval of 0.14 second

ANS: D Because the normal QRS interval is less than 0.12 seconds, the patient's QRS interval of 0.14 seconds indicates that the conduction through the ventricular conduction system is prolonged. The PR interval and QT interval are within normal range and ST segment should be isoelectric (flat).

Which laboratory result for a patient with multifocal premature ventricular contractions (PVCs) is most important for the nurse to communicate to the health care provider? a. Blood glucose of 243 mg/dL b. Serum chloride of 92 mEq/L c. Serum sodium of 134 mEq/L d. Serum potassium of 2.9 mEq/L

ANS: D Hypokalemia increases the risk for ventricular dysrhythmias such as PVCs, ventricular tachycardia, and ventricular fibrillation. The health care provider will need to prescribe a potassium infusion to correct this abnormality. Although the other laboratory values are also abnormal, they are not likely to be the cause of the patient's PVCs and do not require immediate correction.

A patient has the following electrocardiogram (ECG) tracing. Which action would the nurse take? Call a Code Blue. Continue to monitor the patient. Evaluate the patient's electrolyte laboratory results. Document the rhythm as type I second-degree atrioventricular (AV) block.

Continue to monitor the patient. The patient is in a first-degree AV Block, a benign rhythm. The patient is not in a lethal rhythm, so do not call a Code Blue. It is not necessary to evaluate electrolytes. It is not a type I second-degree AV block.

A patient reports a sudden onset of dizziness. The nurse records the following electrocardiogram (ECG) tracing. Which condition is the likely cause of the dizziness? Inner-ear infection Myocardial infarction Decreased cardiac output Rapid metoprolol administration

Decreased cardiac output The patient's rhythm is paroxysmal supraventricular tachycardia (PSVT). Depending on the rate and duration of PSVT, the patient often experiences symptoms related to decreased cardiac output. The cardiac output drops because of decreased ventricular filling time. Although an inner-ear infection can cause dizziness, the ECG tracing is more likely to be the source of the dizziness. The ECG tracing is not indicative of a myocardial infarction. Metoprolol is given to treat hypertension and to decrease the heart rate.

Which action does flecainide have on the heart? Decreases automaticity Accelerates repolarization Decreases impulse conduction Reduces myocardial contractility

Decreases impulse conduction Flecainide is a class IC sodium channel blocker; it decreases impulse conduction. Mexiletine is a class IB sodium channel blocker that accelerates repolarization. β-adrenergic blockers like esmolol decrease the automaticity of the sinoatrial node. Myocardial contractility is reduced with diltiazem, a calcium channel blocker.

The nurse analyzes a patient's telemetry monitor strip and documents a heart rate of 86, regular rhythm, PR interval of 0.24 second, and a QRS interval of 0.10 second. How should the nurse interpret the findings? Type I second-degree atrioventricular (AV) block Type II second-degree AV block Premature ventricular contraction (PVC) First-degree AV block

First-degree AV block In first-degree AV block, there is prolonged duration of AV conduction that lengthens the PR interval to greater than 0.20 second. In type I second-degree AV block, the PR interval continues to lengthen until a QRS complex is blocked. In type II second-degree AV block, the PR interval may be normal or prolonged, but the ventricular rhythm may be irregular, and the QRS usually is usually greater than 0.12 second. PVCs cause an irregular rhythm, and the QRS complex is wide and distorted in shape.

Which observation on a patient's telemetry electrocardiogram (ECG) monitor is a cause for concern? Upright P wave Flat ST segment Upright T wave Prolonged QT interval

Prolonged QT interval Prolonged QT interval is cause for concern. QT disturbance may be caused by drugs, electrolyte imbalances, and changes in heart rate. Upright P wave, flat ST segment, and upright T wave are normal findings.

The patient has atrial fibrillation with a rapid ventricular response. What electrical treatment option does the nurse prepare the patient for? Defibrillation Synchronized cardioversion Automatic external defibrillator (AED) Implantable cardioverter-defibrillator (ICD)

Synchronized cardioversion Rationale: Synchronized cardioversion is planned for a patient with supraventricular tachydysrhythmias (atrial fibrillation with a rapid ventricular response). Defibrillation or AEDs are the treatment of choice to end ventricular fibrillation and pulseless ventricular tachycardia (VT). An ICD is used with patients who have survived sudden cardiac death, have spontaneous sustained VT, and are at high risk for future life-threatening dysrhythmias.

The nurse provides teaching to a group of nursing students about radiofrequency catheter ablation therapy. Which information would the nurse include? The procedure has a high complication rate. The procedure is done before electrophysiological study (EPS). The therapy is considered the nonpharmacologic treatment of choice for severe bradycardic heart rhythms. The therapy uses electrical energy to remove problematic areas of the heart's conduction system.

The therapy uses electrical energy to remove problematic areas of the heart's conduction system. Radiofrequency catheter ablation therapy uses electrical energy to "burn" or ablate areas of the conduction system as definitive treatment of tachydysrhythmias. The ablation procedure is successful with a low complication rate. Ablation therapy is done after EPS has identified the source of the dysrhythmia. The therapy is considered the nonpharmacologic treatment of choice for atrial dysrhythmias, resulting in rapid ventricular rates and atrioventricular (AV) nodal recurrent tachycardia refractory to drug therapy.

Cardioversion is attempted for a patient with atrial flutter and a rapid ventricular response. After delivering 50 joules by synchronized cardioversion, the patient develops ventricular fibrillation. Which action should the nurse take immediately? Administer 250 mL of 0.9% saline solution IV by rapid bolus. Assess the apical pulse, blood pressure, and bilateral neck vein distention. Turn the synchronizer switch to the "off" position and recharge the device. Ask the patient if there is any chest pain or discomfort and administer morphine sulfate.

Turn the synchronizer switch to the "off" position and recharge the device. Rationale: Ventricular fibrillation produces no effective cardiac contractions or cardiac output. If during synchronized cardioversion the patient becomes pulseless or the rhythm deteriorates to ventricular fibrillation, the nurse should turn the synchronizer switch off and initiate defibrillation. Fluids, additional assessment, or treatment of pain alone will not restore an effective heart rhythm.

A patient has pulseless ventricular tachycardia, and cardiopulmonary resuscitation (CPR) is in progress. Which is the first step the nurse would take after a defibrillator is available? Turn on the device. Analyze the rhythm. Apply conductive gel pads. Select proper energy setting.

Turn on the device. The health care team may have to perform defibrillation, and the first step is to turn on the defibrillator. Applying conductive gel pads, energy selection, and rhythm analysis follow.

Defibrillation is indicated for which dysrhythmia? Ventricular fibrillation Uncontrolled atrial fibrillation Ventricular tachycardia with a pulse Third-degree atrioventricular (AV) block

Ventricular fibrillation Defibrillation is always indicated in the treatment of ventricular fibrillation. Drug treatments are normally used in the treatment of uncontrolled atrial fibrillation and for ventricular tachycardia with a pulse (if the patient is stable). Otherwise, synchronized cardioversion is used, as long as the patient has a pulse. Pacemakers are the treatment of choice for third-degree heart block.

Which cardiac dysrhythmia is life threatening and requires immediate intervention? Sinus tachycardia Atrial fibrillation Junctional tachycardia Ventricular fibrillation

Ventricular fibrillation Ventricular fibrillation is a life-threatening dysrhythmia that requires immediate intervention. During ventricular fibrillation, the ventricles are quivering and no longer able to contract to produce effective cardiac output. Because there is no cardiac output, the body is left without oxygenation. Sinus tachycardia requires treatment to slow the rate to 60 to 100 beats/min. Atrial fibrillation requires treatment to convert the rhythm back to a normal sinus rhythm with one atrial contraction for every ventricular contraction. Junctional tachycardia requires no treatment unless it is poorly tolerated.

In the patient with supraventricular tachycardia, which assessment indicates decreased cardiac output? a. Hypertension and dyspnea b. Chest pain and palpitations c. Abdominal distention and tachypnea d. Bounding pulses and a systolic murmur

b Rationale: Manifestations of decreased cardiac output in the patient with supraventricular tachycardia include hypotension, angina, palpitations, and dyspnea.

Important teaching for the patient scheduled for a radiofrequency catheter ablation procedure includes explaining that a. ventricular bradycardia may be induced and treated during the procedure. b. catheter will be placed in both femoral arteries to allow double-catheter use. c. the procedure will destroy areas of the conduction system that are causing rapid heart rhythms. d. general anesthetic will be given to prevent the awareness of any "sudden cardiac death" experiences.

c Rationale: Radiofrequency catheter ablation therapy involves the use of electrical energy to "burn" or ablate areas of the conduction system as definitive treatment of tachydysrhythmias.

The nurse prepares a patient for synchronized cardioversion knowing that cardioversion differs from defibrillation in that a. defibrillation delivers a lower dose of electrical energy. b. cardioversion is a treatment for atrial bradydysrhythmias. c. defibrillation is synchronized to deliver a shock during the QRS complex. d. patients should be sedated if cardioversion is done on a nonemergency basis.

d Rationale: Synchronized cardioversion is the therapy of choice for patients with hemodynamically unstable ventricular or supraventricular tachydysrhythmias. A synchronized circuit in the defibrillator delivers a countershock that is programmed to occur on the R wave of the QRS complex of the electrocardiogram. The synchronizer switch must be turned on when cardioversion is planned. The procedure for synchronized cardioversion is the same as for defibrillation with a few exceptions: If synchronized cardioversion is done on a nonemergency basis, the patient is sedated before the procedure, and the initial energy needed for synchronized cardioversion is less than the energy needed for defibrillation.

The nurse is preparing a patient for an electrocardiogram (ECG). When placing the six unipolar chest leads, at which position should the nurse place the V 6 lead? Fifth intercostal space at the left midaxillary line Fourth intercostal space at the right sternal border Fifth intercostal space at the left midclavicular line Fifth intercostal space at the left anterior axillary line

Fifth intercostal space at the left midaxillary line The V6 lead should be placed at the fifth intercostal space at the left midaxillary line. The V4 lead is placed at the fifth intercostal space at the left midclavicular line. The V5 lead is placed at the fifth intercostal space at the left anterior axillary line. The V1 lead is placed at the fourth intercostal space at the right sternal border.

Which is a disadvantage of centralized monitoring system? What is a disadvantage of a centralized monitoring system? It cannot diagnose dysrhythmias It measures the patients' heart rate remotely It cannot rapidly detect myocardial ischemia It requires continuous observation of patients' ECGs

It requires continuous observation of patients' ECGs A centralized monitoring system is a type of telemetry monitoring system. It requires continuous observation of a group of patients' electrocardiograms at a central location. Centralized monitoring systems observe heart rate and rhythm remotely, at a site distant from the patient. A centralized monitoring system helps to detect dysrhythmias, ischemia, or infarction in patients.

Which electrocardiogram (ECG) characteristic is consistent with ventricular tachycardia (VT)? Unmeasurable rate and rhythm Rate 140 beats/minute; inverted P wave Rate 200 beats/minute; P wave not visible Rate 125 beats/minute; normal QRS complex

Rate 200 beats/minute; P wave not visible VT is associated with a rate of 150 to 250 beats/minute; the P wave normally is not visible. Rate and rhythm are not measurable in ventricular fibrillation. P wave inversion and a normal QRS complex are not associated with VT.

The nurse performs discharge teaching for a patient with an implantable cardioverter-defibrillator (ICD). Which statement by the patient indicates that further teaching is needed? "The device may set off the metal detectors in an airport." "My family needs to keep up to date on how to perform CPR." "I should not stand next to antitheft devices at the exit of stores." "I can expect redness and swelling of the incision site for a few days."

"I can expect redness and swelling of the incision site for a few days." Rationale: Patients should be taught to report any signs of infection at incision site (e.g., redness, swelling, drainage) or fever to their primary care providers immediately. Teach patients to inform TSA airport security of the presence of the ICD because it may set off metal detectors. If a handheld screening wand is used, it should not be placed directly over the ICD. Teach patients to avoid standing near antitheft devices in doorways of stores and public buildings and to walk through them at a normal pace. Caregivers should learn cardiopulmonary resuscitation.

The nurse is doing discharge teaching with the patient who received an implantable cardioverter-defibrillator (ICD) in the left side. Which statement by the patient indicates that further teaching is required? "I will call the cardiologist if my ICD fires." "I cannot fly because it will damage the ICD." "I cannot move my left arm until it is approved." "I cannot drive until my cardiologist says it is okay."

"I cannot fly because it will damage the ICD." Rationale: The patient statement that flying will damage the ICD indicates misunderstanding about flying. The patient should be taught to inform TSA security screening agents at the airport about the ICD because it may set off the metal detector and if a hand-held screening wand is used, it should not be placed directly over the ICD. The other options indicate the patient understands the teaching.

The nurse provides discharge teaching to a patient who received an implantable cardioverter-defibrillator (ICD) in the left side of the chest. Which statement made by the patient indicates the need for further teaching? "I cannot drive until my cardiologist gives me permission." "I cannot lift my left arm above my shoulder until it is approved." "I cannot fly because the security devices at the airport will damage the ICD." "I cannot stand near antitheft devices in doorways of stores and public buildings."

"I cannot fly because the security devices at the airport will damage the ICD." The patient statement that flying will damage the ICD indicates misunderstanding about flying. The patient should be taught that the Transportation Security Administration (TSA) should be informed about the ICD because it may set off the metal detector, and if a hand-held screening wand is used it should not be placed directly over the ICD. "I cannot drive until my cardiologist says it is okay," "I cannot move my left arm until it is approved," and "I cannot stand near antitheft devices in doorways of stores and public buildings" indicate that the patient understands the teaching.

The nurse is reviewing discharge instructions with a patient who received an implantable cardioverter-defibrillator (ICD). Which patient statement indicates the need for further teaching? "I should avoid flying for three years." "I should avoid direct blows to the ICD site." "I should avoid standing near antitheft devices in doorways." "I should avoid large magnets and strong electromagnetic fields."

"I should avoid flying for three years." Flying is not contraindicated in a patient with ICD. To ensure safety, the patient should inform airport security about the presence of the ICD at the airport because it may set off the metal detector. The patient should avoid direct blows to the ICD site to reduce pressure at the site. Electric and magnetic signals from antitheft devices and strong electromagnetic fields can affect ICD functioning.

A patient with a newly inserted permanent pacemaker receives discharge instructions from the nurse. Which patient statement indicates that further teaching is required? Correct "I should avoid using microwave ovens." "I should avoid standing near antitheft devices." "I should avoid direct blows to the pacemaker site." "I should avoid close proximity to high-output electric generators."

"I should avoid using microwave ovens." Microwaves do not interfere with a pacemaker's function and can be used safely. Electric signals from antitheft devices can affect pacemaker functioning. The patient should avoid direct blows to the pacemaker site to reduce pressure at the site. Electric signals from high-output electric generators can move the pacemaker from its position and affect its functioning.

A patient develops third-degree heart block and reports feeling chest pressure and shortness of breath. Which instructions should the nurse provide to the patient before initiating emergency transcutaneous pacing? "The device will convert your heart rate and rhythm back to normal." "The device uses overdrive pacing to slow the heart to a normal rate." "The device is inserted through a large vein and threaded into your heart." "The device delivers a current through your skin that can be uncomfortable."

"The device delivers a current through your skin that can be uncomfortable." Rationale: Before initiating transcutaneous pacing therapy, it is important to tell the patient what to expect. The nurse should explain that the muscle contractions created by the pacemaker when the current passes through the chest wall are uncomfortable. Pacing for complete heart block will not convert the heart rhythm to normal. Overdrive pacing is used for very fast heart rates. Transcutaneous pacing is delivered through pacing pads adhered to the skin.

In which order would the nurse take the steps for applying telemetry monitoring electrodes? Monitor for artifact. Affix the electrodes. Clip excessive hair with scissors. Gently rub the skin with dry gauze.

1. Clip excessive hair with scissors. 2. Gently rub the skin with dry gauze. 3. Affix the electrodes. 4. Monitor for artifact. The first step during the measurement of an electrocardiogram (ECG) is to remove excess hair on the skin. The presence of hair may not facilitate the proper adherence of electrodes to the skin. This could interfere with the recording of the cardiac impulses. It is followed by wiping the skin with alcohol to remove dirt and oil, and gently rubbing with gauze until the skin becomes pale pink. Wiping of the skin is followed by attaching the electrodes to the patient at the designated areas. Artifact may occur for various reasons. This indicates deformity of baseline and waveforms on electrocardiograph.

A patient is diagnosed with pulseless ventricular tachycardia. In which order would the nurse perform the steps of defibrillation? Check to see that the synchronizer switch is turned off. Apply conductive materials to the chest. Turn the defibrillator on and select the proper energy level. Deliver the charge by depressing buttons on both paddles simultaneously. Charge the defibrillator using the button on the defibrillator or the paddles. Position the paddles firmly on the chest wall over the conductive material. Call and look to see that everyone is "all clear."

1. Turn the defibrillator on and select the proper energy level. 2. Check to see that the synchronizer switch is turned off. 3. Apply conductive materials to the chest. 4. Charge the defibrillator using the button on the defibrillator or the paddles. 5. Position the paddles firmly on the chest wall over the conductive material. 6. Call and look to see that everyone is "all clear." Defibrillation is the treatment of choice for pulseless ventricular tachycardia. The nurse would first turn the defibrillator on and select the proper energy level, then check to see that the synchronizer switch is turned off. The nurse then would apply conductive materials to the chest and charge the defibrillator using the button on the defibrillator or the paddles. The nurse would then position the paddles firmly on the chest wall over the conductive material. The nurse would then check to see that everyone is "all clear." Finally, the nurse would deliver the charge by depressing buttons on both paddles simultaneously.

A patient experiencing syncope is prescribed the head-up tilt-test. In which order would the steps of the procedure be performed? Place the patient on a table supported by a belt across the torso and feet. Tilt the table 60 to 80 degrees. Keep the patient in the upright position for 20 to 60 minutes. Record the electrocardiogram and heart rate continuously, and measure the BP every three minutes. Obtain a baseline electrocardiogram, BP, and heart rate in the horizontal position.

1.Place the patient on a table supported by a belt across the torso and feet. 2.Obtain a baseline electrocardiogram, BP, and heart rate in the horizontal position. 3.Tilt the table 60 to 80 degrees. 4.Keep the patient in the upright position for 20 to 60 minutes. 5.Record the electrocardiogram and heart rate continuously, and measure the BP every three minutes. The head-up tilt-test is a procedure used to determine the cause of fainting spells in a patient. It is a simple, noninvasive, and informative test. The first step is to place the patient on a table supported by a belt across the torso and feet. Next, a baseline electrocardiogram, BP, and heart rate are obtained in the horizontal position. Next, the table is tilted 60 to 80 degrees, and the patient is kept in this upright position for 20 to 60 minutes. The electrocardiogram and heart rate are recorded continuously, and BP is measured every three minutes.

When computing a heart rate from the electrocardiography (ECG) tracing, the nurse counts 15 of the small blocks between the R waves of a patient whose rhythm is regular. What does the nurse calculate the patient's heart rate to be? 60 beats/min 75 beats/min 100 beats/min 150 beats/min

100 beats/min Rationale: Because each small block on the ECG paper represents 0.04 seconds, 1500 of these blocks represents 1 minute. By dividing the number of small blocks (15, in this case) into 1500, the nurse can calculate the heart rate in a patient whose rhythm is regular (in this case, 100).

After the nurse gives IV atropine to a patient with symptomatic type 1, second-degree atrioventricular (AV) block, which finding indicates that the drug has been effective? a. Increase in the patient's heart rate b. Increase in strength of peripheral pulses c. Decrease in premature atrial contractions d. Decrease in premature ventricular contractions

ANS: A Atropine will increase the heart rate and conduction through the AV node. Because the drug increases electrical conduction, not cardiac contractility, the quality of the peripheral pulses is not used to evaluate the drug effectiveness. The patient does not have premature atrial or ventricular contractions.

A 20-yr-old patient has a mandatory electrocardiogram (ECG) before participating on a college soccer team. The patient is found to have sinus bradycardia, rate 52 and blood pressure (BP) 114/54 mm Hg. The student denies any health problems. What action by the nurse is appropriate? a. Allow the student to participate on the soccer team. b. Refer the student to a cardiologist for further testing. c. Tell the student to stop playing immediately if any dyspnea occurs. d. Obtain more detailed information about the student's family health history.

ANS: A In an aerobically trained individual, sinus bradycardia is normal. The student's normal BP and negative health history indicate that there is no need for a cardiology referral or for more detailed information about the family's health history. Dyspnea during an aerobic activity such as soccer is normal.

Which action will the nurse include in the plan of care for a patient who was admitted with syncopal episodes of unknown origin? a. Explain the association between dysrhythmias and syncope. b. Instruct the patient to call for assistance before getting out of bed. c. Teach the patient about the need to avoid caffeine and other stimulants. d. Tell the patient about the benefits of implantable cardioverter-defibrillators.

ANS: B A patient with fainting episodes is at risk for falls. The nurse will plan to minimize the risk by having assistance whenever the patient is up. The other actions may be needed if dysrhythmias are found to be the cause of the patient's syncope but are not appropriate for syncope of unknown origin.

Which action by a new registered nurse (RN) who is orienting to the telemetry unit indicates a good understanding of the treatment of heart dysrhythmias? a. Prepares defibrillator settings at 360 joules for a patient whose monitor shows asystole. b. Injects IV adenosine (Adenocard) over 2 seconds for a patient with supraventricular tachycardia. c. Turns the synchronizer switch to the "on" position before defibrillating a patient with ventricular fibrillation. d. Gives the prescribed dose of diltiazem (Cardizem) to a patient with new-onset type II second-degree AV block.

ANS: B Adenosine must be given over 1 to 2 seconds to be effective. The other actions indicate a need for more teaching about treatment of heart dysrhythmias. The RN should hold the diltiazem until discussing it with the health care provider. The treatment for asystole is immediate CPR. The synchronizer switch should be "off" when defibrillating

A patient has a sinus rhythm and a heart rate of 72 beats/min. The nurse determines that the PR interval is 0.24 seconds. What action should the nurse take? a. Notify the health care provider immediately. b. Document the finding and monitor the patient. c. Give atropine per agency dysrhythmia protocol. d. Prepare the patient for temporary pacemaker insertion.

ANS: B First-degree atrioventricular block is asymptomatic and requires ongoing monitoring because it may progress to more serious forms of heart block. The rate is normal, so there is no indication that atropine is needed. Immediate notification of the health care provider about an asymptomatic rhythm is not necessary.

A patient has ST segment changes that suggest an acute inferior wall myocardial infarction. Which lead would be the most useful for monitoring the patient? a. I b. II c. V2 d. V6

ANS: B Leads II, III, and AVF reflect the inferior area of the heart and the ST segment changes. Lead II will best capture any electrocardiographic changes that indicate further damage to the myocardium. The other leads do not reflect the inferior part of the myocardial wall and will not provide data about further ischemic changes in that area.

A patient is apneic and has no palpable pulses. The heart monitor shows sinus tachycardia, rate 132. What action should the nurse take next? a. Perform synchronized cardioversion. b. Start cardiopulmonary resuscitation (CPR). c. Give atropine per agency dysrhythmia protocol. d. Apply supplemental O2 via non-rebreather mask.

ANS: B The patient's manifestations indicate pulseless electrical activity, and the nurse should immediately start CPR. The other actions would not be of benefit to this patient.

A patient's heart monitor shows a pattern of undulations of varying contours and amplitude with no measurable ECG pattern. The patient is unconscious, apneic, and pulseless. Which action should the nurse take first? a. Give epinephrine (Adrenalin) IV. b. Perform immediate defibrillation. c. Prepare for endotracheal intubation. d. Ventilate with a bag-valve-mask device.

ANS: B The patient's rhythm and assessment indicate ventricular fibrillation and cardiac arrest; the initial action should be to defibrillate. If a defibrillator is not immediately available or is unsuccessful in converting the patient to a better rhythm, begin chest compressions. The other actions may also be appropriate but not first.

Which nursing action can the registered nurse (RN) delegate to experienced unlicensed assistive personnel (UAP) working as telemetry technicians on the cardiac care unit? a. Decide whether a patient's heart rate of 116 requires urgent treatment. b. Observe heart rhythms for multiple patients who have telemetry monitoring. c. Monitor a patient's level of consciousness during synchronized cardioversion. d. Select the best lead for monitoring a patient admitted with acute coronary syndrome.

ANS: B UAP serving as telemetry technicians can monitor heart rhythms for individuals or groups of patients. Nursing actions such as assessment and choice of the most appropriate lead based on ST segment elevation location require RN-level education and scope of practice.

A patient with supraventricular tachycardia who is alert and has a blood pressure of 110/66 mm Hg is being prepared for cardioversion. Which action should the nurse take? a. Turn the synchronizer switch to the "off" position. b. Give a sedative before cardioversion is implemented. c. Set the defibrillator/cardioverter energy to 360 joules. d. Provide assisted ventilations with a bag-valve-mask device.

ANS: B When a patient has a nonemergency cardioversion, sedation is used just before the procedure. The synchronizer switch is turned "on" for cardioversion. The initial level of joules for cardioversion is low (e.g., 50). Assisted ventilations are not indicated for this patient.

A patient on the telemetry unit develops atrial flutter, rate 150, with associated dyspnea and chest pain. Which action that is included in the agency dysrhythmia protocol should the nurse do first? a. Obtain a 12-lead electrocardiogram (ECG). b. Notify the health care provider of the change in rhythm. c. Give supplemental O2 at 2 to 3 L/min via nasal cannula. d. Assess the patient's blood pressure and discomfort level.

ANS: C Because this patient has dyspnea and chest pain in association with the new rhythm, the nurse's initial actions should be to address the patient's airway, breathing, and circulation (ABC) by starting with O2 administration. The other actions are also important and should be implemented rapidly.

A patient has a junctional escape rhythm on the monitor. What heart rate should the nurse expect the patient to have? a. 15 to 20 b. 20 to 40 c. 40 to 60 d. 60 to 100

ANS: C If the sinoatrial (SA) node does not discharge, the atrioventricular (AV) node will automatically discharge at the normal rate of 40 to 60 beats/min. The slower rates are typical of the bundle of His and Purkinje system and may be seen with failure of both the SA and AV node to discharge. The normal SA node rate is 60 to 100 beats/min.

A 19-year-old student comes to the student health center at the end of the semester stating, "My heart is skipping beats." An electrocardiogram (ECG) shows occasional unifocal premature ventricular contractions (PVCs). What action should the nurse take next? a. Insert an IV catheter for emergency use. b. Start supplemental O2 at 2 to 3 L/min via nasal cannula. c. Ask the patient about current stress level and caffeine use. d. Have the patient taken to the nearest emergency department (ED).

ANS: C In a patient with a normal heart, occasional PVCs are a benign finding. The timing of the PVCs suggests stress or caffeine as possible etiologic factors. The patient is hemodynamically stable, so there is no indication that the patient needs supplemental O2, an IV, or to be seen in the ED.

Which information will the nurse include when teaching a patient with atrial flutter who is scheduled for a radiofrequency catheter ablation? a. The procedure stimulates the growth of new pathways between the atria. b. The procedure uses cold therapy to stop the formation of the flutter waves. c. The procedure uses electrical energy to destroy areas of the conduction system. d. The procedure prevents or minimizes the patient's risk for sudden cardiac death.

ANS: C Radiofrequency catheter ablation therapy uses electrical energy to "burn" or ablate areas of the conduction system as definitive treatment of atrial flutter (i.e., restore normal sinus rhythm) and tachydysrhythmias. All other statements about the procedure are incorrect.

A patient reports dizziness and shortness of breath for several days. During heart monitoring in the emergency department (ED), the nurse obtains the following electrocardiographic (ECG) tracing. How does the nurse interpret this heart rhythm? a. Junctional escape rhythm b. Accelerated idioventricular rhythm c. Third-degree atrioventricular (AV) block d. Sinus rhythm with premature atrial contractions

ANS: C The inconsistency between the atrial and ventricular rates and the variable PR interval indicate that the rhythm is third-degree AV block. Sinus rhythm with PACs will have a normal rate and consistent PR intervals with occasional PACs. An accelerated idioventricular rhythm will not have visible P waves.

A patient's heart monitor shows sinus rhythm, rate 64. The PR interval is 0.18 seconds at 1:00 AM, 0.22 seconds at 2:30 PM, and 0.28 seconds at 4:00 PM. Which action should the nurse take first? a. Place the transcutaneous pacemaker pads on the patient. b. Give atropine sulfate 1 mg IV per agency dysrhythmia protocol. c. Hold the scheduled metoprolol (Lopressor) and call the health care provider. d. Document the patient's rhythm and PR measurements in the medical record.

ANS: C The patient has progressive first-degree atrioventricular (AV) block, and the -blocker should be held until discussing the drug with the health care provider. Documentation is appropriate later. The patient with first-degree AV block usually is asymptomatic; if the patient became symptomatic, a pacemaker or atropine may be used.

A patient who reports a "racing" heart and feeling "anxious" comes to the emergency department. The nurse places the patient on a heart monitor and obtains the following electrocardiographic (ECG) tracing. Which action should the nurse take next? a. Prepare to perform electrical cardioversion. b. Have the patient perform the Valsalva maneuver. c. Obtain the patient's vital signs including O2 saturation. d. Prepare to give a -blocker medication to slow the heart rate.

ANS: C The patient has sinus tachycardia, which may have multiple causes, such as pain, dehydration, anxiety, and myocardial ischemia. Further assessment is needed before determining the treatment. Vagal stimulation or -blockade may be used after further assessment of the patient. Electrical cardioversion is a treatment for some tachydysrhythmias but not sinus tachycardia.

Which action by a nurse caring for a patient after an implantable cardioverter-defibrillator (ICD) insertion indicates a need for more teaching about the care of patients with ICDs? a. The nurse administers amiodarone (Cordarone) to the patient. b. The nurse helps the patient fill out the application for obtaining a Medic Alert device. c. The nurse encourages the patient to do active range-of-motion exercises for all extremities. d. The nurse teaches the patient that sexual activity can be resumed when the incision is healed.

ANS: C The patient should avoid moving the arm on the ICD insertion site until healing has occurred to prevent displacement of the ICD leads. The other actions by the new nurse are appropriate for this patient.

The nurse needs to quickly estimate the heart rate for a patient with a regular heart rhythm. Which method will be fastest to use? a. Count the number of large squares in the R-R interval and divide by 300. b. Print a 1-minute electrocardiogram (ECG) strip and count the number of QRS complexes. c. Use the 3-second markers to count the number of QRS complexes in 6 seconds and multiply by 10. d. Calculate the number of small squares between one QRS complex and the next and divide into 1500.

ANS: C Using the 3-second markers to count the number of QRS complexes in 6 seconds and multiplying by 10 is the quickest way to determine the ventricular rate for a patient with a regular rhythm. The other methods are accurate but take longer.

What should the nurse measure to determine whether there is a delay in impulse conduction through the patient's ventricles? a. P wave b. Q wave c. PR interval d. QRS complex

ANS: D The QRS complex represents ventricular depolarization. The P wave represents the depolarization of the atria. The PR interval represents depolarization of the atria, atrioventricular node, bundle of His, bundle branches, and the Purkinje fibers. The Q wave is the first negative deflection following the P wave and should be narrow and short.

The nurse obtains a rhythm strip on a patient who has had a myocardial infarction and makes the following analysis: no visible P waves, PR interval not measurable, ventricular rate of 162, R-R interval regular, QRS complex wide and distorted, and QRS duration of 0.18 second. How should the nurse interpret this cardiac rhythm? a. Atrial flutter b. Sinus tachycardia c. Ventricular fibrillation d. Ventricular tachycardia

ANS: D The absence of P waves, wide QRS, rate greater than 150 beats/min, and the regularity of the rhythm indicate ventricular tachycardia. Atrial flutter is usually regular, has a narrow QRS configuration, and has flutter waves present representing atrial activity. Sinus tachycardia has P waves. Ventricular fibrillation is irregular and does not have a consistent QRS duration.

A patient who was admitted with a myocardial infarction has a 45-second episode of ventricular tachycardia, then converts to sinus rhythm with a heart rate of 98 beats/min. Which action should the nurse take next? a. Immediately notify the health care provider. b. Document the rhythm and continue to monitor the patient. c. Prepare for synchronized cardioversion per agency protocol. d. Prepare to give IV amiodarone per agency dysrhythmia protocol.

ANS: D The burst of sustained ventricular tachycardia indicates that the patient has significant ventricular irritability, and antidysrhythmic medication administration is needed to prevent further episodes. The nurse should notify the health care provider after the medication is started. Cardioversion is not indicated given that the patient has returned to a sinus rhythm. Documentation and continued monitoring are not adequate responses to this situation.

The nurse has received change-of-shift report about the following patients on the progressive care unit. Which patient should the nurse see first? a. A patient with atrial fibrillation, rate 88 and irregular, who has a dose of warfarin (Coumadin) due b. A patient with second-degree atrioventricular (AV) block, type 1, rate 60, who is dizzy when ambulating c. A patient who is in a sinus rhythm, rate 98 and regular, recovering from an elective cardioversion 2 hours ago d. A patient whose implantable cardioverter-defibrillator (ICD) fired twice today and has a dose of amiodarone (Cordarone) due

ANS: D The frequent firing of the ICD indicates that the patient's ventricles are very irritable. The priority is to assess the patient and give the amiodarone. The other patients can be seen after the amiodarone is given.

A patient develops sinus bradycardia at a rate of 32 beats/min, has a blood pressure (BP) of 80/42 mm Hg, and reports feeling faint. Which action should the nurse take? a. Reposition the patient on the left side. b. Have the patient perform the Valsalva maneuver. c. Give the scheduled dose of diltiazem (Cardizem). d. Apply the transcutaneous pacemaker (TCP) pads.

ANS: D The patient is experiencing symptomatic bradycardia and treatment with TCP is appropriate. Calcium channel blockers will further decrease the heart rate and the diltiazem should be held. The Valsalva maneuver will further decrease the rate. Repositioning on the left side may decrease cardiac output and blood pressure further.

The nurse evaluates that discharge teaching about the management of a new permanent pacemaker has been effective when the patient states a. "It will be several weeks before I can return to my usual activities." b. "I will avoid cooking with a microwave oven or being near one in use." c. "I will notify the airlines when I make a reservation that I have a pacemaker." d. "I won't lift the arm on the pacemaker side until I see the health care provider."

ANS: D The patient is instructed to avoid lifting the arm on the pacemaker side above the shoulder to avoid displacing the pacemaker leads. The patient should notify airport security about the presence of a pacemaker before going through the metal detector, but there is no need to notify the airlines when making a reservation. Microwave oven use does not affect the pacemaker. The insertion procedure involves minor surgery that will have a short recovery period.

A patient with chest pain experiences a heart rate of 200 beats/min and BP of 80/50 mm Hg. The electrocardiogram shows absent P waves. Which IV medication would the nurse expect will be prescribed? Digoxin Atropine Adenosine Vasopressin

Adenosine Paroxysmal supraventricular tachycardia (PSVT) is a dysrhythmia starting in an ectopic focus anywhere above the bifurcation of the bundle of His. The symptoms associated with PSVT include hypotension, palpitations, dyspnea, and angina. In PSVT, the heart rate will be greater than 180 beats/min and the electrocardiogram will often show a hidden P wave. IV adenosine is the standard drug for paroxysmal supraventricular tachycardia. Digoxin, atropine, and vasopressin are not prescribed for paroxysmal supraventricular tachycardia. Digoxin is used in the treatment of atrial fibrillation. Atropine is used in the treatment of junctional escape rhythm. Vasopressin is used in the treatment of asystole.

The nurse is caring for a patient who is clinically stable while experiencing monomorphic ventricular tachycardia. Which intervention would be included on the patient's treatment plan? Perform defibrillation. Administer amiodarone. Administer a vasopressor. Initiate cardiopulmonary resuscitation.

Administer amiodarone. Amiodarone is an antiarrhythmic agent that corrects various atrial and ventricular dysrhythmias. Since the patient is clinically stable, amiodarone can be used. Defibrillation is not indicated for a patient that is clinically stable. There is no need to administer a vasopressor to a clinically stable patient. Cardiopulmonary resuscitation is not needed for a patient with a pulse who is breathing.

patient on a cardiac unit is shivering. Which finding would the nurse expect to see on the patient's electrocardiogram (ECG) tracing? Artifact Asystole Atrial flutter Junctional dysrhythmia

Artifact An artifact is a distortion of the baseline and waveforms seen on the ECG. If the patient is shivering or shows any muscle activity, accurate interpretation of the heart rhythm is difficult, and artifacts can occur on the monitor. Asystole is the absence of all cardiac electrical activity. Atrial flutter occurs in chronic lung disease or hypertension. Junctional dysrhythmias are associated with an electrolyte imbalance or rheumatic heart disease.

The nurse responds to a cardiac monitor alarm and notes that atrial flutter has developed on a patient's electrocardiogram (ECG). The patient is awake and talking. Which action would the nurse take? Assess the patient for dyspnea. Prepare for synchronized cardioversion. Initiate cardiopulmonary resuscitation (CPR). Place the patient in the Trendelenburg position.

Assess the patient for dyspnea. Because the patient is awake and responsive, the next action is to assess the patient for stability and the possible cause of the dysrhythmia. The nurse should focus the assessment on vital signs, such as BP, heart rate, and respiratory status. Synchronized cardioversion would be an option only if the patient were unstable. Initiating CPR is appropriate only for a patient that is in respiratory or cardiac arrest. The Trendelenburg position is inappropriate for a patient with atrial flutter.

The nurse is caring for a patient 24 hours after the patient underwent pacemaker insertion surgery. Which action will be included on the postoperative plan of care? Changing the surgical dressing as needed Encouraging range-of-motion exercises of the involved arm Assessing the incision for any redness, swelling, or discharge Applying wet-to-dry dressings every four hours to the insertion site

Assessing the incision for any redness, swelling, or discharge After pacemaker insertion, it is important for the nurse to observe signs of infection by assessing for any redness, swelling, or discharge from the incision site. The dressing is kept on and dry until removed 24 hours postoperatively. It is important for the patient to limit activity of the involved arm to minimize pacemaker lead displacement.

The nurse is caring for a patient who is 24 hours after pacemaker insertion. Which nursing intervention is appropriate at this time? Reinforcing the pressure dressing as needed Encouraging range-of-motion exercises of the involved arm Assessing the incision for any redness, swelling, or discharge Applying wet-to-dry dressings every 4 hours to the insertion site

Assessing the incision for any redness, swelling, or discharge Rationale: After pacemaker insertion, it is important for the nurse to observe signs of infection by assessing for any redness, swelling, or discharge from the incision site. The nonpressure dressing is kept dry until removed, usually 24 hours postoperatively. It is important for the patient to limit activity of the involved arm to minimize pacemaker lead displacement.

A patient states, "I feel tired all the time, and I struggle with activities of daily living." The patient's heart rate is 120 beats/minute. Which rhythm does the nurse anticipate observing on the electrocardiogram tracing? Asystole Atrial fibrillation Sinus bradycardia Ventricular fibrillation

Atrial fibrillation Atrial fibrillation can reduce cardiac output due to the loss of atrial kick and a rapid ventricular rate, causing a reduced exercise tolerance and an elevated heart rate above 100 beats/minute. The patient will not have a pulse and will not be responsive if the rhythm is ventricular fibrillation or asystole. A patient with sinus bradycardia will have a heart rate less than 60 beats/minute.

A patient has an atrial rate of 450 beats per minute and a ventricular rate of 150 beats per minute. Which condition is the patient likely experiencing? Atrial flutter Atrial fibrillation Ventricular fibrillation Premature ventricular contractions

Atrial fibrillation Atrial fibrillation is characterized by alterations in electrical conductivity of the atrium. The atrial rate is as high as 350 beats per minute to 600 beats per minute. Atrial fibrillation with controlled ventricular response is observed at a ventricular rate of 60 beats per minute to 100 beats per minute. A ventricular rate above 100 beats per minute is considered as atrial fibrillation with uncontrolled ventricular response. The atrial rate and ventricular rate in atrial flutter occur at the ratio of 2:1. The atrial rate is observed between 200 beats per minute to 350 beats per minute and the ventricular rate occurs at 150 beats per minute. Ventricular fibrillation elicits dysrhythmia and P wave and QRS complex cannot be predicted. Premature ventricular contractions elicit dysrhythmia with deflections in the heart rate.

A patient's electrocardiogram (ECG) now shows no P waves, fine and wavy lines between the QRS complexes, QRS complexes that measure 0.08 sec, and QRS complexes that occur irregularly with a rate of 120 beats/minute. Which interpretation of the rhythm would the nurse make? Atrial fibrillation Sinus tachycardia Ventricular fibrillation Ventricular tachycardia

Atrial fibrillation Atrial fibrillation is represented on the cardiac monitor by irregular R-R intervals and small fibrillatory (f) waves. There are no normal P waves because the atria are not contracting truly, just fibrillating. Sinus tachycardia is a sinus rate above 100 beats/minute with normal P waves. Ventricular fibrillation is seen on the ECG without a visible P wave; an unmeasurable heart rate, PR, or QRS; and an irregular and chaotic rhythm. Ventricular tachycardia is seen as three or more premature ventricular contractions (PVCs) that have distorted QRS complexes with a regular or irregular rhythm; the P wave is usually buried in the QRS complex without a measurable PR interval.

The nurse observes no P waves on the patients monitor strip. There are fine, wavy lines between the QRS complexes. The QRS complexes measure 0.08 seconds (narrow), but they occur irregularly with a rate of 120 beats/min. What does the nurse determine the rhythm to be? Sinus tachycardia Atrial fibrillation Ventricular fibrillation Ventricular tachycardia

Atrial fibrillation Rationale: Atrial fibrillation is represented on the cardiac monitor by irregular R-R intervals and small fibrillatory (f) waves. There are no normal P waves because the atria are not truly contracting, just fibrillating. Sinus tachycardia is a sinus rate above 100 beats/min with normal P waves. Ventricular fibrillation is seen on the ECG without a visible P wave; an unmeasurable heart rate, PR or QRS; and the rhythm is irregular and chaotic. Ventricular tachycardia is seen as three or more premature ventricular contractions that have distorted QRS complexes with regular or irregular rhythm, and the P wave is usually buried in the QRS complex without a measurable PR interval.

Which cardiac activity is absent when a permanent pacemaker fails to capture? Spontaneous atrial activity Atrial or ventricular contraction Spontaneous ventricular activity Excitability during the cardiac cycle

Atrial or ventricular contraction Failure to capture occurs when the electrical charge to the myocardium is insufficient to produce atrial or ventricular contraction. Failure to sense occurs when the pacemaker fails to recognize spontaneous atrial or ventricular activity, and it fires inappropriately. This can result in the pacemaker firing during the excitable period of the cardiac cycle, resulting in ventricular tachycardia.

Which electrocardiogram (ECG) findings are consistent with type II second-degree atrioventricular (AV) block? Select all that apply. Atrial rhythm is regular. Ventricular rate is regular. PR intervals are consistent. QRS complex is less than 0.12 second. There are more P waves than QRS complexes.

Atrial rhythm is regular. PR intervals are consistent. There are more P waves than QRS complexes Type II second-degree AV block (Mobitz II) is a more critical type of heart block that requires early recognition and intervention. There is more than one P wave for each QRS complex, usually in the ratio of 2:1, 3:1, or more. There is no progressive lengthening of the PR interval, which remains the same throughout, with the exception of the dropped beat(s). Atrial rhythm is regular, while ventricular rhythm may be irregular. QRS complex is greater than 0.12 second because of the presence of bundle branch block.

A patient is hospitalized for treatment of symptoms associated with a junctional escape rhythm, including a heart rate (HR) of 45 beats/min. Which medication would the nurse expect to be prescribed for this patient? Atropine Propranolol Amiodarone Cardioversion

Atropine A junctional escape rhythm is a delayed heartbeat that originates from an ectopic focus somewhere in the atrioventricular junction, not from the atrium. If a patient has symptoms with a junctional escape rhythm, atropine can be used; it will increase the patient's HR. Propranolol and amiodarone would not be appropriate for this patient because they cause bradycardia. Cardioversion should not be used for junctional dysrhythmias.

The nurse is caring for a patient who is experiencing symptomatic sinus bradycardia. Which drugs are used to treat this rhythm? Select all that apply. Atropine Dopamine Adenosine Metoprolol Epinephrine

Atropine Dopamine Epinephrine Sinus bradycardia is a condition in which the sinoatrial node elicits a heartbeat at a rate of less than 80 beats/min, sinus bradycardia is associated with hypotension, weakness, dizziness, and shortness of breath. It can be treated by the administration of atropine, an anticholinergic drug. Sympathomimetic drugs like dopamine and epinephrine are administered if atropine is ineffective. β-blockers like adenosine and metoprolol are used in the treatment of sinus tachycardia.

Which term is used for the property of heart cells to initiate an impulse spontaneously and continuously? Excitability Automaticity Conductivity Contractibility

Automaticity Automaticity is the heart's ability to initiate an impulse spontaneously and continuously. Excitability is the ability to be electrically stimulated. Conductivity is the ability to transmit an impulse along a membrane in an orderly manner. Contractility is the ability to respond mechanically to an impulse.

The nurse provides discharge teaching to a patient who underwent pacemaker implantation in the left chest. Which instructions would the nurse include? Select all that apply. Showering is permitted as long as care is taken to pat dry the incision site. Avoid direct blows to the incision site. Raise the left arm above the head frequently, to promote circulation. Monitor pulse and inform the health care provider if the heart rate drops below the predetermined rate. Microwave ovens are safe to use.

Avoid direct blows to the incision site. Monitor pulse and inform the health care provider if the heart rate drops below the predetermined rate. Microwave ovens are safe to use. The patient should avoid direct blows to the incision site for safety reasons. The patient should monitor the pulse and inform the cardiologist if it drops below the predetermined rate. Microwave ovens are safe to use and do not interfere with the function of the pacemaker. The incision site should be kept dry for four days after implantation, so showering should be avoided. The patient should avoid lifting the arm above the shoulder until approved by the cardiologist because it might affect the function of the pacemaker.

Which term is used for the property of the cardiac cell responding mechanically to an impulse? Excitability Contractility Automaticity Conductivity

Contractility Cardiac cells have different properties that are associated with the stimulation and formation of impulse. The property of the cardiac cell to respond mechanically to an impulse is called contractility. The property of the cardiac cell to be electrically stimulated is called excitability. The property of the cardiac cell to initiate an impulse spontaneously and continuously is called automaticity. The property of the cardiac cell to transmit an impulse along a membrane in an orderly manner is called conductivity.

Which property of heart cells is defined as the ability of the heart to respond mechanically to an impulse? Automaticity Excitability Conductivity Contractility

Contractility Contractility enables the cardiac cells to respond mechanically to an impulse. Automaticity provides the ability to initiate an impulse spontaneously and continuously. Excitability enables the cardiac cells to be electrically stimulated. Conductivity allows transmission of an impulse along a membrane in an orderly manner.

The nurse is caring for a patient who develops atrial fibrillation. Which treatments may be included the patient's treatment plan? Select all that apply. Atropine injection Electrical cardioversion Anticoagulation therapy Implantable cardioverter-defibrillator (ICD) Radiofrequency catheter ablation

Electrical cardioversion Anticoagulation therapy Radiofrequency catheter ablation Electrical cardioversion converts the atrial fibrillation into normal sinus rhythm. If a patient is in atrial fibrillation for more than 48 hours, anticoagulation therapy with warfarin will be required for three to four weeks. This should commence before the cardioversion and has to be continued for several weeks after successful cardioversion. For patients with drug-refractory atrial fibrillation or those who do not respond to electrical conversion, radiofrequency catheter ablation may be done. Atropine increases the heart rate and would not be prescribed for a patient with atrial fibrillation. The ICD is not appropriate for patients with atrial fibrillation.

A patient with a heart rate of 120 beats/minute is prescribed an antidysrhythmic drug. Which diagnostic test would the nurse anticipate the patient having? Holter monitoring Exercise treadmill testing Electrophysiologic study (EPS) Signal-averaged electrocardiogram (ECG)

Electrophysiologic study (EPS) An ESP identifies the causes of heart blocks, tachydysrhythmias, bradydysrhythmias, and syncope. It can also locate accessory pathways and determine the effectiveness of antidysrhythmic drugs. A Holter monitor is used to record ECG when the patient is ambulatory and performing daily activities. Exercise treadmill testing evaluates the patient's heart rhythm during exercise. Signal-averaged ECG identifies the late potential if the patient is at risk of developing serious arrhythmias.

While explaining temporary pacemakers to a group of nursing students, which information would the nurse include? Select all that apply. All temporary pacemakers are transvenous. Epicardial pacing involves attaching leads to the epicardium during heart surgery. A transvenous pacemaker consists of leads that are threaded into the left ventricle. Epicardial pacemakers are inserted in emergency departments and critical care units in emergency situations. A transvenous pacemaker is attached to an external power source. The placement of a transcutaneous pacemaker is noninvasive; it is a temporary procedure.

Epicardial pacing involves attaching leads to the epicardium during heart surgery. A transvenous pacemaker is attached to an external power source. The placement of a transcutaneous pacemaker is noninvasive; it is a temporary procedure. Epicardial pacing wires are inserted into the epicardial wall of the heart during cardiac surgery. The wires are brought through the chest wall and can be connected to a pulse generator, if needed. Four wires are placed through the chest wall of the patient: two wires from the atrium and two wires from the ventricles. These four wires are connected to the temporary pacemaker, and pacing thresholds are set for each patient. Placement of the transcutaneous pacemaker is noninvasive and a temporary method until more permanent treatment is sought. Transcutaneous pacemakers use electrical stimulation that is delivered through the skin via external electrode pads connected to an external pacemaker (a defibrillator with pacemaker functions). Temporary pacemakers include transcutaneous pacemakers, transvenous pacemakers, and epicardial pacemakers. With transvenous pacemakers, a pacing catheter is inserted percutaneously into the right ventricle, where it gets connected to the endocardium near the ventricular septum. It is connected to a small external pulse generator by electrode wires. Epicardial pacemakers are inserted during open-chest surgery.

Which factors contribute to artifact on a patient's telemetry monitor? Disabled automaticity Stimulation of the vagus nerve fibers Electrodes placed in the incorrect lead Excessive hair under the electrode pads

Excessive hair under the electrode pads One reason that artifact is seen on the monitor is when leads and electrodes are not secure. Electrode pads may not be secure if there is excessive hair under the pads, the skin is oily, or diaphoresis is present. Disabled automaticity would cause an atrial dysrhythmia. Stimulation of the vagus nerve fibers causes a decrease in heart rate, not artifact. Electrodes placed in the incorrect place will measure electricity in a different plane of the heart and may have a different wave form than expected.

A patient's electrocardiogram (ECG) shows prolonged PR interval, normal P waves, and normal QRS complexes. The patient is asymptomatic and has a normal heart rate and a regular rhythm. Which type of atrioventricular (AV) block is present? First-degree AV block Third-degree AV block Type I second-degree AV block Type II second-degree AV block

First-degree AV block First-degree AV block conducts every impulse to the ventricles with prolonged AV conduction time. In first-degree AV block, heart rate is normal and heart rhythm is regular. The ECG of a patient with first-degree AV block shows normal P wave, prolonged PR interval, and the normal shaped QRS complex. Third-degree AV block is suspected if no impulses are conducted from atria to ventricles. Type I second-degree AV block is suspected if the rhythm on the ECG appears as grouped beats. Type II second-degree AV block is suspected if the ventricular rhythm is irregular.

The patient has a potassium level of 2.9 mEq/L, and the nurse obtains the following measurements on the rhythm strip: Heart rate of 86 with a regular rhythm, the P wave is 0.06 seconds (sec) and normal shape, the PR interval is 0.24 seconds, and the QRS is 0.09 seconds. How should the nurse document this rhythm? First-degree AV block Second-degree AV block Premature atrial contraction (PAC) Premature ventricular contraction (PVC)

First-degree AV block Rationale: In first-degree atrioventricular (AV) block, there is prolonged duration of AV conduction that lengthens the PR interval above 0.20 seconds. In type I second-degree AV block, the PR interval continues to increase in duration until a QRS complex is blocked. In type II, the PR interval may be normal or prolonged, the ventricular rhythm may be irregular, and the QRS is usually greater than 0.12 seconds. PACs cause an irregular rhythm with a different-shaped P wave than the rest of the beats, and the PR interval may be shorter or longer. PVCs cause an irregular rhythm, and the QRS complex is wide and distorted in shape.

A patient who reports dizziness and shortness of breath is admitted to the emergency department. The following is the patient's electrocardiogram (ECG) tracing. Which item listed on the patient's plan of care would the nurse question? Epinephrine 12-lead ECG IV adenosine Transcutaneous pacing

IV adenosine IV adenosine is the drug of choice to convert paroxysmal supraventricular tachycardia (PSVT) to a normal sinus rhythm. Adenosine acts in opposition to adrenaline and would be harmful to a patient in third-degree atrioventricular (AV) block. The use of drugs such as dopamine and epinephrine is a temporary measure to increase heart rate (HR) and support BP until temporary pacing is started. The nurse would expect to receive a prescription for a 12-lead ECG. Transcutaneous pacing is used to increase the HR until a transvenous or permanent pacemaker can be placed.

Which is a disadvantage of a centralized monitoring system? It cannot diagnose dysrhythmias. It measures the patients' heart rate remotely. It cannot rapidly detect myocardial ischemia. It requires continuous observation of patients' electrocardiograms (ECGs).

It requires continuous observation of patients' electrocardiograms (ECGs). A centralized monitoring system is a type of telemetry monitoring system. It requires continuous observation of a group of patients' electrocardiograms at a central location. Centralized monitoring systems observe heart rate and rhythm remotely, at a site distant from the patient. A centralized monitoring system helps to detect dysrhythmias, ischemia, or infarction in patients.

A patient who underwent a cardioverter-defibrillator implant (ICD) procedure is being discharged from the health care facility. Which instructions would the nurse give the patient and caregiver? Select all that apply. Keep the incision dry for four days after insertion or as instructed. Exercise the arm on the incision side with a full range of motion. Avoid large magnets and strong electromagnetic fields. Avoid air travel until the cardiologist gives permission. Inform the cardiologist if the implanted ICD fires.

Keep the incision dry for four days after insertion or as instructed. Avoid large magnets and strong electromagnetic fields. Inform the cardiologist if the implanted ICD fires. The incision should be kept dry for as many days as instructed by the cardiologist. The patient should avoid metal detectors, large magnets, and strong electromagnetic fields. If the ICD fires, then the patient should inform the cardiologist or contact emergency medical service. The patient should not lift the arm on the side of the ICD until approved by the cardiologist. Air travel is not restricted; however, the patient should not drive unless cleared by the cardiologist.

Which are the most common electrocardiograph leads selected for continuous monitoring? Select all that apply. Lead II Lead III Lead V1 Lead V6 Lead aVR

Lead II Lead V1 An electrocardiogram is a graphical representation of the electrical impulses produced in the heart. It involves the use of 12 leads. Lead II and lead V1 are commonly used for continuous monitoring of the cardiac impulses. The use of lead III, lead V6, and lead aVR are also a part of the normal 12-lead electrocardiogram but are not usually used for continuous monitoring.

The patient is admitted with acute coronary syndrome (ACS). The ECG shows ST-segment depression and T-wave inversion. What should the nurse know that this indicates? Myocardia injury Myocardial ischemia Myocardial infarction Normal pacemaker function.

Myocardial ischemia Rationale: The ST depression and T wave inversion on the ECG of a patient diagnosed with ACS indicate myocardial ischemia from inadequate supply of blood and oxygen to the heart. Myocardial injury is identified with ST-segment elevation. Myocardial infarction is identified with ST-segment elevation and a widened and deep Q wave. A pacemaker's presence is evident on the ECG by a spike leading to depolarization and contraction.

A patient with a heart rate of 180 beats/minute has a regular heart rhythm, normal P waves, and normal PR intervals. Which QRS complex shape would the nurse expect to see on the patient's electrocardiogram (ECG) tracing? Normal Not measurable Wide and distorted Abnormally shaped

Normal A normal P wave and normal PR interval in ECG and the heart rate of 180 beats/minute indicates sinus tachycardia. The patient with sinus tachycardia generally shows normal QRS complexes. The QRS complex will not be measurable in the patient with ventricular fibrillation. The QRS complex will be wide and distorted or abnormally shaped in the ECG of a patient who has ventricular tachycardia or premature ventricular contractions. The P wave in these patients will not usually be visible, and the PR interval will not be measurable.

Which electrocardiogram (ECG) characteristics will be present if a patient is experiencing unifocal premature ventricular contractions (PVCs)? Select all that apply. Normal T wave P wave that is not visible PR interval that is not measurable Wide and distorted QRS complex PVCs that have the same shape

Normal T wave PR interval that is not measurable Wide and distorted QRS complex PVCs that have the same shape A PVC is a contraction that results from an ectopic focus in the ventricles. In other words, the QRS complex occurs prematurely. The T wave is generally normal. The PR interval is not measurable. The QRS complex is wide and distorted in shape compared with a QRS complex coming down a normal conduction pathway. PVCs that arise from the same foci appear the same in shape and are called unifocal PVCs. The P wave is rarely visible and is usually hidden in the PVC.

A patient with acute coronary syndrome has continuous electrocardiogram (ECG) monitoring in place. The nurse analyzes the patient's rhythm strip and notes a normal atrial rate, regular atrial rhythm, normal-shaped P waves, widened QRS complexes, and two P waves preceding every QRS complex. There is no progressive lengthening of PR intervals. Which action would the nurse take? Apply a transcutaneous pacemaker. Perform synchronized cardioversion. Administer amiodarone IV infusion. Observe for symptoms of decreased cardiac output (CO).

Observe for symptoms of decreased cardiac output (CO). The rhythm is a type II second-degree atrioventricular (AV) block (i.e., Mobitz II). The rhythm is identified by noting P waves that are nonconducted without progressive PR lengthening. This usually occurs when a block in one of the bundle branches is present. On conducted beats, the PR interval is constant. Type II second-degree AV block is a more serious type of block in which a certain number of impulses from the sinoatrial (SA) node are not conducted to the ventricles. This occurs in ratios of 2:1, 3:1, and so on (i.e., two P waves to one QRS complex, three P waves to one QRS complex). The nurse should assess for bradycardia, hypotension, and angina. If the patient becomes symptomatic, a temporary pacemaker may be needed. Cardioversion would not be useful in treating a heart block. Amiodarone is an antidysrhythmic that would likely increase the heart block.

Which instruction would the nurse include in discharge teaching for a patient with an implantable cardioverter-defibrillator (ICD)? Select all that apply. Avoid or limit air travel. Obtain and wear a Medic Alert ID device at all times. Avoid large magnets and strong electromagnetic fields. Avoid lifting the arm on ICD side above the shoulder until approved. Do not walk through antitheft devices in doorways of stores and public buildings.

Obtain and wear a Medic Alert ID device at all times. Avoid large magnets and strong electromagnetic fields. Avoid lifting the arm on ICD side above the shoulder until approved. Patients with ICDs need to obtain and wear a Medic Alert ID device at all times, avoid large magnets and strong electromagnetic fields because they may interfere with ICD function, and avoid lifting their arms on the ICD side above their shoulders until approved. These patients do not need to avoid air travel. They can walk through antitheft devices at a normal pace but should not stand next to them.

Symptomatic second-degree heart block type 2 can be managed with which procedure? Ablation Cardioversion Pacemaker insertion Cardiac catheterization

Pacemaker insertion The treatment for symptomatic second-degree atrioventricular (AV) block type 2 is a permanent pacemaker. An ablation, cardioversion, and cardiac catheterization would not effectively treat atrioventricular (AV) block type 2.

A patient is sitting up in bed and talking while the bedside electrocardiogram tracing shows the rhythm below. Which action would the nurse take first? Perform rapid defibrillation. Palpate the patient for a pulse. Take the patient's BP. Start cardiopulmonary resuscitation (CPR).

Palpate the patient for a pulse. The rhythm strip characterizes ventricular tachycardia (VT). Therefore the first step is to palpate the pulse. The treatment for VT with a pulse differs greatly than for VT without a pulse. If the VT is monomorphic and the patient has a pulse and has preserved left-ventricular function, IV drugs are used. VT without a pulse is a life-threatening situation; rapid defibrillation and CPR are the first lines of treatment in this scenario. The nurse can take the patient's BP if a pulse is present.

The nurse is reviewing the electrocardiograms of four patients. Which conclusion would the nurse make about these electrocardiograms? Patient A has atrial flutter. Patient C has sinus bradycardia. Patient D has sinus tachycardia. Patient B has accelerated junctional rhythm.

Patient B has accelerated junctional rhythm. An accelerated junctional rhythm has a rate in the range of 61 to 100 beats/min and abnormal and inverted P waves. This describes Patient B. The atrial rate of 400 beats/min indicates atrial fibrillation, not atrial flutter. The heart rate of 150 beats/min, normal P wave, and normal PR interval indicate sinus tachycardia, not sinus bradycardia. The heart rate of 50 beats/min, normal P wave, and normal PR interval indicate sinus bradycardia, not sinus tachycardia.

The nurse is reviewing prescriptions of four patients. Which inference would the nurse make? Patient A has second-degree heart block. Patient B has third-degree heart block. Patient C has junctional escape rhythm. Patient D has sinus bradycardia.

Patient C has junctional escape rhythm. A patient with junctional escape rhythm shows a heart rate in the range of 40 to 60 beats/minute. Atropine increases the ventricular rate and can effectively improve the junctional escape rhythm in Patient C. Heart block in patient A would be worsened by amiodarone. Heart block in Patient B would be worsened by adenosine and would be treated with a temporary pacemaker or chronotropic drugs. Sinus bradycardia in Patient D would not be amenable to cardioversion and, if symptomatic, would require chronotropic drugs or a pacemaker.

The nurse reviews the electrocardiograms (ECGs) of four patients. Which conclusion would the nurse make from these findings? Patient B: There is organized electrical activity of the atria. Patient D: There are recurring regular, sawtooth-shaped flutter waves. Patient C: The discharge rate from the sinoatrial (SA) node is decreased. Patient A: The sinoatrial (SA) node is functioning as the pacemaker of the heart.

Patient D: There are recurring regular, sawtooth-shaped flutter waves Variable and not measurable PR intervals with an atrial rate of 200 to 350 beats/min are characteristics of atrial flutter in Patient D. The inverted P wave and PR interval of less than 0.12 seconds indicates a junctional dysrhythmia in Patient A. Replacement of P waves with chaotic, fibrillatory waves in the ECG and the atrial rate of 500 beats/min indicates atrial fibrillation in Patient B. The normal P wave and the heart rate of 150 beats/min indicate sinus tachycardia in Patient C.

The electrocardiogram (ECG) monitor of a patient in the cardiac care unit after myocardial infarction (MI) indicates ventricular fibrillation. Which action would the nurse take immediately? Perform synchronized cardioversion. Administer IV amiodarone. Perform cardiopulmonary resuscitation (CPR). Prepare for insertion of a temporary transvenous pacemaker.

Perform cardiopulmonary resuscitation (CPR). Treatment consists of immediate initiation of CPR and advanced cardiac life support (ACLS) with the use of defibrillation and definitive drug therapy (e.g., epinephrine, vasopressin). There should be no delay in using a defibrillator once available. Amiodarone, cardioversion, and temporary pacemakers are not used to treat ventricular fibrillation.

The nurse reviews a patient's electrocardiogram (ECG) tracing and notes a heart rate of 82 and an irregular rhythm. Which interpretation of the findings would the nurse make? Sinus tachycardia Junctional dysrhythmia Premature atrial contractions Paroxysmal supraventricular tachycardia

Premature atrial contractions When premature atrial contractions are present, the result is an irregular rhythm. In sinus tachycardia, the patient's heart rate is 101 to 200 beats/min, and the cardiac rhythm is regular. A heart rate of 40 to 180 beats/min with regular cardiac rhythm is observed in patients with junctional dysrhythmias. A heart rate of 150 to 220 beats/min with regular cardiac rhythm is observed in patients with paroxysmal supraventricular tachycardia.

A patient's electrocardiogram (ECG) tracing shows occasional wide and distorted QRS complexes. Which rhythm does this finding indicate? Sinus tachycardia Ventricular fibrillation Junctional dysrhythmias Premature ventricular contractions

Premature ventricular contractions Premature ventricular contractions are caused by premature impulses originating from the ventricles of the heart and not from the sinoatrial node. This causes the QRS complex to be wide and distorted. The QRS wave appears normal in patients with sinus tachycardia. The QRS wave is not measurable in the ECG of a patient with ventricular fibrillation. The QRS complex will be normal in the ECG of patient with junctional dysrhythmias.

The nurse finds a patient with a weak, irregular pulse who is apneic and unarousable, and the cardiac monitor shows torsades de pointes. Which action would the nurse take? Prepare the patient for intubation. Prepare the patient for defibrillation. Prepare to administer amiodarone. Prepare the patient for insertion of temporary pacemaker.

Prepare the patient for defibrillation. This patient is experiencing symptomatic ventricular tachycardia with torsades de pointes which requires defibrillation to return to a normal rhythm. Oxygen delivery would follow with bag-valve ventilation followed by intubation if necessary. Amiodarone is indicated for treatment of ventricular tachycardia but is not a priority over defibrillation. Temporary pacemaker application is done for the patient with a symptomatic bradycardia.

A patient informs the nurse of experiencing syncope. Which prioitiy nursing action should the nurse anticipate in the patient's subsequent diagnostic workup? Preparing to assist with a head-up tilt-test Assessing the patient's knowledge of pacemakers Administering an IV dose of a β-adrenergic blocker Teaching the patient about antiplatelet aggregators

Preparing to assist with a head-up tilt-test Rationale: In patients without structural heart disease, the head-up tilt-test is a common component of the diagnostic workup after episodes of syncope. IV β-blockers are not indicated, although an IV infusion of low-dose isoproterenol may be started in an attempt to provoke a response if the head-up tilt-test did not have a response. Addressing pacemakers is premature and inappropriate at this stage of diagnosis. Patient teaching surrounding antiplatelet aggregators is not directly relevant to the patient's syncope at this time.

A patient is experiencing atrial flutter. Which treatment will be included in the patient's plan of care? Carotid massage The Maze procedure Anticholinergic drugs Radiofrequency catheter ablation

Radiofrequency catheter ablation Radiofrequency catheter ablation is the most effective technique for the treatment of atrial flutter. It is performed in the electrophysiology study laboratory and involves the introduction of a catheter in the right atrium. The tissue is targeted and destroyed by the application of low-voltage, high-frequency electrical impulses. The destruction of the tissue results in a normal sinus rhythm. Anticholinergic drugs would increase the heart rate and are not appropriate to treat atrial flutter. Carotid massage is a maneuver for vagal stimulation and is generally used for treating paroxysmal supraventricular tachycardia. The Maze procedure is performed to treat atrial fibrillations that are refractory to drugs, electrical conversion, and radiofrequency catheter ablation.

Which advice would a nurse give to a caregiver with regards to a patient who has an implantable cardioverter-defibrillator (ICD)? Select all that apply. Restrict air travel. Report any signs of infection at incision site. Restrict MRI scan. If the ICD fires more than once, contact the emergency response system (ERS). Restrict the lifting of the arm on the ICD side above the shoulder until approved.

Report any signs of infection at incision site. Restrict MRI scan. If the ICD fires more than once, contact the emergency response system (ERS). Restrict the lifting of the arm on the ICD side above the shoulder until approved. The patient with an ICD should immediately report any signs of infection, such as redness, swelling, drainage, or fever. The patient should not undergo MRI scan unless the ICD is approved as MRI-safe. The patient should be instructed to contact ERS if the ICD fires more than once. Also, the arm on the ICD side should not be lifted above the shoulder unless approved by the cardiologist. Air travel is not restricted. However, while traveling, the patient should inform the airport security personnel about the presence of the ICD because it may set off the metal detector.

The nurse suspects that a patient is experiencing myocardial infarction (MI). Which component of the patient's electrocardiogram (ECG) is important for the nurse to analyze to make this determination? P wave PR interval ST segment Heart rate

ST segment The typical ECG change seen during myocardial injury is ST segment elevation. The P wave represents time for the passage of the electrical impulse through the atrium; there are no specific changes that occur with MI. A prolonged PR interval is indicative of a heart block and is usually not associated with MI. A specific change in heart rate is not associated with MI.

Which electrocardiogram (ECG) characteristic in a patient with acute coronary syndrome suggests myocardial ischemia? Sinus arrhythmia Normal T wave Pathologic Q wave ST segment depression

ST segment depression Typical ECG changes that are seen in myocardial ischemia include ST segment depression and/or T wave inversion. The typical ECG change seen during an acute myocardial infarction is ST segment elevation. Depression of the ST segment and T wave inversion occurs in response to an inadequate supply of blood and oxygen, which causes an electrical disturbance in the myocardial cells. In sinus arrhythmia, the conduction pathway is the same as that in sinus rhythm, but the SA node fires irregularly. It is common in healthy adults. Pathologic Q waves may be seen in patients with a myocardial infarction. ST segment elevation may be seen in myocardial injury.

Which type of dysrhythmia is associated with a fever? Atrial fibrillation Sinus tachycardia Sinus bradycardia Junctional tachycardia

Sinus tachycardia A fever may cause tachycardia with a decrease in cardiac output and hypotension. Atrial fibrillation can cause significant problems for a patient with complex disease processes, but it is not related to fever. Bradycardia and junctional tachycardia are not typical results of fever.

The nurse has obtained this rhythm strip from her patient's monitor. What should the nurse document this rhythm indicates? Sinus tachycardia Sinus bradycardia Ventricular fibrillation Ventricular tachycardia

Sinus tachycardia Rationale: This rhythm strip shows sinus tachycardia because the rate on this strip is above 101 beats/min, and it displays normal P wave, PR interval, and QRS complex. Sinus bradycardia would look similar to sinus tachycardia but with a rate less than 60 beats/min. Ventricular fibrillation does not have a measureable heart rate, PR interval, or QRS. The P wave is not visible, and the rhythm is irregular and chaotic. Ventricular tachycardia has a rate of 150 to 250 beats/min, with a regular or irregular rhythm and P waves occurring independently of the QRS complex.

A patient's electrocardiogram (ECG) shows a heart rate of 150 beats/minute and a normal P wave preceding each QRS complex. Which interpretation would the nurse make of these findings? Atrial fibrillation Sinus tachycardia Ventricular fibrillation Premature atrial contractions

Sinus tachycardia Sinus tachycardia includes a heart rate of 101 beats to 180 beats per minute. The electrocardiographic study of sinus tachycardia shows a normal P wave preceding each QRS complex with normal time and duration. In atrial fibrillation, the P waves are chaotic and fibrillatory and the QRS complex is normal. The electrocardiographic study of ventricular fibrillation elicits the absence of P waves, and the PR interval and QRS interval cannot be measured. In premature atrial contraction, there are distorted P waves in the ECG.

Which interpretation would the nurse make of this electrocardiogram (ECG) rhythm? Sinus tachycardia Sinus bradycardia Ventricular fibrillation Ventricular tachycardia

Sinus tachycardia This rhythm strip shows sinus tachycardia because the rate on this strip is above 101, and it displays normal P wave, PR interval, and QRS complex. Sinus bradycardia would look similar to sinus tachycardia but with a rate less than 60 beats/minute. Ventricular fibrillation does not have a measurable heart rate, PR interval, or QRS; the P wave is not visible, and the rhythm is irregular and chaotic. Ventricular tachycardia has a rate of 150 to 250 beats/minute, with a regular or irregular rhythm and P waves occurring independently of the QRS complex.

Which statement made by the student nurse indicates the need for additional teaching about electrocardiograms (ECGs)? Leads I, II, and III are bipolar. Leads aVR, aVL, and aVF are unipolar. Six bipolar leads (V1 through V6) measure the electrical forces in the horizontal plane. A 12-lead ECG may show changes that suggest an electrolyte imbalance.

Six bipolar leads (V1 through V6) measure the electrical forces in the horizontal plane. The six leads that measure the electrical forces in the horizontal plane are unipolar leads, not bipolar leads. Leads I, II, and III are bipolar in nature and indicate that they possess both positive and negative charges. Leads aVR, aVL, and aVF are unipolar. A 12-lead ECG may show changes suggesting structural changes, conduction disturbances, damage, electrolyte imbalance, or drug toxicity.

The nurse is providing care for a patient with type I second-degree atrioventricular (AV) block. Which statements about this type of rhythm are accurate? Select all that apply. A pacemaker is the only viable treatment. Some P waves are conducted to the ventricles. The P waves are not conducted to the ventricles. If the patient is symptomatic, atropine or a pacemaker may be needed. It may result from drugs such as digoxin.

Some P waves are conducted to the ventricles. If the patient is symptomatic, atropine or a pacemaker may be needed. It may result from drugs such as digoxin. It is generally transient and well tolerated. Type I second-degree heart block refers to AV conduction that is intermittently blocked. Therefore some P waves are conducted to the ventricle and some are not. Symptomatic patients may need atropine or a transcutaneous pacemaker. This type of heart block may result from drugs such as digoxin, and the condition is transient and well tolerated. If the patient is asymptomatic, no treatments may be needed. Third-degree block is often called complete heart block because no atrial impulses are conducted through the AV node to the ventricles.

The nurse is providing care for a patient with type I second-degree atrioventricular (AV) block. Which statements about this type of rhythm are accurate? Select all that apply. A pacemaker is the only viable treatment. Some P waves are conducted to the ventricles. The P waves are not conducted to the ventricles. If the patient is symptomatic, atropine or a pacemaker may be needed. It may result from drugs such as digoxin. It is generally transient and well tolerated.

Some P waves are conducted to the ventricles. If the patient is symptomatic, atropine or a pacemaker may be needed. It may result from drugs such as digoxin. It is generally transient and well tolerated. Type I second-degree heart block refers to AV conduction that is intermittently blocked. Therefore some P waves are conducted to the ventricle and some are not. Symptomatic patients may need atropine or a transcutaneous pacemaker. This type of heart block may result from drugs such as digoxin, and the condition is transient and well tolerated. If the patient is asymptomatic, no treatments may be needed. Third-degree block is often called complete heart block because no atrial impulses are conducted through the AV node to the ventricles.

Which statement is accurate regarding the role of the autonomic nervous system in impulse formation? Stimulation of the parasympathetic nerves increases cardiac contractility. Stimulation of the vagus nerve causes a decreased rate of firing of the sinoatrial (SA) node. Stimulation of the sympathetic nerves decreases atrioventricular (AV) node impulse conduction. Stimulation of the vagus nerve causes increased impulse conduction of the AV node.

Stimulation of the vagus nerve causes a decreased rate of firing of the sinoatrial (SA) node. The autonomic nervous system plays an important role in the rate of impulse formation, the speed of conduction, and the strength of cardiac contraction. Stimulation of the vagus nerve causes a decreased rate of firing of the sinoatrial node. Stimulation of the parasympathetic system decreases cardiac contractility. Stimulation of the sympathetic nerves increases atrioventricular node impulse conduction. Stimulation of the vagus nerve decreases impulse conduction of the atrioventricular node. Vagus nerve fibers of the parasympathetic nervous system and nerve fibers of the sympathetic nervous system are the components of the autonomous nervous system that affect the heart rate.

A patient is hospitalized following a three-day history of heart palpitations and dizziness. The patient's electrocardiogram (ECG) shows the following rhythm. For which condition is this patient at risk? Stroke Type 2 diabetes Myocardial infarction (MI) Sudden cardiac death

Stroke A risk for atrial fibrillation is clot formation in the atria caused by altered blood flow through the heart. If the clot forms on the left side of the heart, the risk is for the clot traveling to the brain, causing a stroke or an embolism to other arteries in the body. If the clot forms on the right side of the heart, then it can travel to the lungs, causing a pulmonary embolism. The risk for an embolus is particularly high when the patient converts back to a normal sinus rhythm. To reduce the risk for clot formation in the heart, most patients with chronic atrial fibrillation are on some type of anticoagulation. Coumadin (warfarin) is often prescribed. Atrial fibrillation does not increase the risk for diabetes, MI, or sudden cardiac death.

A patient with paroxysmal supraventricular tachycardia (PSVT) who is receiving IV adenosine becomes hemodynamically unstable. Which intervention would the nurse expect to be included in the patient's immediate treatment plan? β-adrenergic blocker Calcium channel blocker Catheter ablation therapy Synchronized cardioversion

Synchronized cardioversion PSVT is a dysrhythmia starting in an ectopic focus anywhere above the bifurcation of the bundle of His. The standard drug of choice to treat PSVT is IV adenosine. Sometimes the drug therapy is ineffective, and the patient becomes hemodynamically unstable. For patients who are unresponsive to treatment, synchronized cardioversion is used. Synchronized cardioversion is low energy shock, which uses a sensor to deliver electricity that is synchronized with the peak of the QRS complex. Drug therapy is not effective for such patients. β-adrenergic blockers and calcium channel blockers do not improve PSVT. Catheter ablation therapy is used in patients with Wolff-Parkinson-White syndrome who have recurring PSVT.

A patient develops atrial flutter with a rapid ventricular response. The nurse anticipates that which treatment will be prescribed? Defibrillation Synchronized cardioversion Automatic external defibrillator (AED) Implantable cardioverter-defibrillator (ICD)

Synchronized cardioversion Synchronized cardioversion is the therapy of choice for patients with ventricular tachydysrhythmias (e.g., ventricular tachycardia [VT] with a pulse) or supraventricular tachydysrhythmias (e.g., atrial flutter with a rapid ventricular response). Defibrillation or AEDs are the treatment of choice for ventricular fibrillation and pulseless VT. An ICD is used with patients who have survived sudden cardiac death (SCD), have spontaneous sustained VT, or who are at high risk of future life-threatening dysrhythmias.

A patient is experiencing third-degree AV block. Which electrocardiogram (ECG) findings are characteristic of this rhythm? Select all that apply. The PR interval is variable. The P wave has a normal shape. The ventricular rate is irregular. The atrial rate is more than 100 beats/min. The atrial and ventricular rhythms are regular but unrelated.

The PR interval is variable. The P wave has a normal shape. The atrial and ventricular rhythms are regular but unrelated. A third-degree AV block is often called a complete heart block because no atrial impulses are conducted through the AV node to the ventricles. In such situations, the atria and ventricles beat independently because the AV node is completely blocked to the sinus impulse and, therefore, it is not conducted to the ventricles. One of the characteristics of a third-degree heart block is that the P waves have no association with the QRS complexes and appear throughout the QRS waveform. The P wave has a normal shape. The atrial and ventricular rhythms are regular, but these are not related to each other. The atrial rate is usually a sinus rate of 60 to 100 beats/min.

Which electrocardiogram (ECG) characteristics would the nurse use to interpret that a patient is in normal sinus rhythm (NSR)? Select all that apply. The R-R intervals are relatively consistent. One P wave precedes each QRS complex. Four to five complexes occur in a six-second strip. The ST segment is higher than PR interval. The T wave is upright.

The R-R intervals are relatively consistent. One P wave precedes each QRS complex. The T wave is upright. The consistency of the R-R interval indicates regular rhythm. A normal P wave before each complex indicates that the impulse originated in the SA node. The T wave is upright and represents ventricular repolarization. The number of complexes in a six-second strip is multiplied by 10 to approximate the heart rate; therefore a normal heart rate (60 to 100) is six to ten complexes in a six-second strip. Four to five complexes in a six-second strip would calculate to a heart rate of 40 to 50, which is bradycardia. An elevation of the ST segment is a sign of cardiac ischemia and is not a characteristic of NSR.

The nurse presents information about electrocardiogram (ECG) changes associated with myocardial infarction (MI) to a group of nursing students. Which information would the nurse include? Select all that apply. The T wave will be normal. The ST segment will be elevated. A physiologic Q wave is normally very short and narrow. The pathologic Q wave will eventually disappear from the ECG. A pathologic Q wave that develops during MI is wide and deep.

The ST segment will be elevated. A physiologic Q wave is normally very short and narrow. A pathologic Q wave that develops during MI is wide and deep. The typical ECG changes associated with MI are ST segment elevation, pathologic Q wave, and T wave inversion. ST segment elevation is considered significant if it is 1 mm or more above the isoelectric line in at least two contiguous leads. A physiologic Q wave is the first negative deflection following the P wave. It is normally very short and narrow. A pathologic Q wave that develops during MI is wide (greater than 0.03 seconds in duration) and deep (greater than or equal to 25% of the height of the R wave). The pathologic Q wave may remain on the ECG indefinitely.

After synchronized cardioversion, a patient's electrocardiogram (ECG) tracing reveals the following. Which interpretation would the nurse make? The cardioversion was successful. The cardioversion will need to be repeated. The ECG tracing indicates hyperkalemia. The patient is in an accelerated junctional rhythm.

The cardioversion was successful. The patient has converted to a normal sinus rhythm (NSR). The cardioversion was successful and does not need to be repeated. Hyperkalemia is characterized by a peaked T wave. The T wave in this tracing is normal. Accelerated junctional rhythm is characterized by an absent P wave and inverted P wave before or following the QRS complex.

Which statement describes the electrical activity represented by the PR interval on an electrocardiogram (ECG)? The length of time it takes to depolarize the atrium The length of time it takes for the atria to depolarize and repolarize The length of time for the electrical impulse to travel from the sinoatrial (SA) node to the Purkinje fibers The length of time it takes for the electrical impulse to travel from the SA node to the atrioventricular (AV) node

The length of time for the electrical impulse to travel from the sinoatrial (SA) node to the Purkinje fibers The electrical impulse in the heart must travel from the SA node through the AV node and into the Purkinje fibers for synchronous atrial and ventricular contraction to occur. When measuring the PR interval (the time from the beginning of the P wave to the beginning of the QRS), the nurse is identifying the length of time it takes for the electrical impulse to travel from the SA node to the Purkinje fibers. The P wave represents the length of time it takes for the impulse to travel from the SA node through the atrium, causing depolarization of the atria (atrial contraction). Atrial repolarization occurs during ventricular depolarization and is hidden by the QRS complex. The length of time it takes for the electrical impulse to travel from the SA node to the AV node is the flat line between the end of the P wave and the beginning of the Q wave on the ECG and is not usually measured.

Which statement best describes the electrical activity of the heart represented by measuring the PR interval on the electrocardiogram (ECG)? The length of time it takes to depolarize the atrium. The length of time it takes for the atria to depolarize and repolarize. The length of time for the electrical impulse to travel from the sinoatrial (SA) node to the Purkinje fibers. The length of time it takes for the electrical impulse to travel from the sinoatrial (SA) node to the atrioventricular (AV) node.

The length of time for the electrical impulse to travel from the sinoatrial (SA) node to the Purkinje fibers. Rationale: The electrical impulse in the heart must travel from the SA node through the AV node and into the Purkinje fibers in order for synchronous atrial and ventricular contraction to occur. When measuring the PR interval (the time from the beginning of the P wave to the beginning of the QRS), the nurse is identifying the length of time it takes for the electrical impulse to travel from the SA node to the Purkinje fibers. The P wave represents the length of time it takes for the impulse to travel from the SA node through the atrium, causing depolarization of the atria (atrial contraction). Atrial repolarization occurs during ventricular depolarization and is hidden by the QRS complex. The length of time it takes for the electrical impulse to travel from the SA node to the AV node is the flat line between the end of the P wave and the beginning of the Q wave on the ECG and is not usually measured.

The nurse obtains a prescription for adenosine to treat a patient who is having paroxysmal supraventricular tachycardia (PSVT). Which information would the nurse use in administering the medication? The medication half-life is 10 minutes. The medication decreases conduction through the atrioventricular (AV) node. It must be given through a central venous access device (CVAD). It is given as an IV push (IVP) over one to two minutes.

The medication decreases conduction through the atrioventricular (AV) node. Adenosine stops PSVT by slowing the conduction through the AV node. It may be given through a peripheral IV, but the site should be as close to the heart as possible because of its short half-life of 10 seconds. A brief period of asystole is common. Adenosine is given as an IVP over one to two seconds and then immediately followed by a rapid 20 mL normal saline flush.

Which conditions are possible causes of the artifact in a patient's electrocardiogram (ECG) tracing? Select all that apply. The patient is shivering. The patient has dry skin. The conductive gel is moist. Electrical interference is present. The leads and electrodes are not secure.

The patient is shivering. Electrical interference is present. The leads and electrodes are not secure. Muscle activity caused by shivering of the patient, electrical interference, or loose leads and electrodes can cause distorted baseline and waveforms called an artifact on the ECG. Oily skin is wiped dry with alcohol to prepare the patient for ECG. Electrodes may have to be replaced if conductive gel has dried out.

A patient undergoing treatment for dysrhythmia is provided with a Holter monitor. Which information would the nurse include in the patient teaching about the test? Select all that apply. The patient should activate the monitor when experiencing symptoms. The patient should record activities and any symptoms in a diary. The monitor records the electrocardiogram (ECG) when the patient is ambulatory. New technology using smart phone apps can obtain and save ECG recordings. The monitor records the ECG when the patient performs daily activities.

The patient should record activities and any symptoms in a diary. The monitor records the electrocardiogram (ECG) when the patient is ambulatory. New technology using smart phone apps can obtain and save ECG recordings. The monitor records the ECG when the patient performs daily activities. The patient should keep a diary and record activities and any symptoms. The Holter monitor continuously records the ECG while the patient is ambulatory and performing daily activities. New technology using smart phone apps can obtain and save ECG recordings and even detect some dysrhythmias. The patient would not wait to activate the monitor when having symptoms, which the Holter monitor records continuously. Event monitors are activated only when having symptoms.

The nurse determines there is artifact on the patient's telemetry monitor. Which factor should the nurse assess for that could correct this issue? Disabled automaticity Electrodes in the wrong lead Too much hair under the electrodes Stimulation of the vagus nerve fibers

Too much hair under the electrodes Rationale: Artifact is caused by muscle activity, electrical interference, or insecure leads and electrodes that could be caused by excessive chest wall hair. Disabled automaticity would cause an atrial dysrhythmia. Electrodes in the wrong lead will measure electricity in a different plane of the heart and may have a different wave form than expected. Stimulation of the vagus nerve fibers causes a decrease in heart rate, not artifact.

A patient who is hospitalized with heart failure is experiencing chest pain and shortness of breath and has a BP of 70/40 mm Hg. The patient has the following electrocardiogram (ECG) tracing. Which intervention would the nurse expect will be included in the patient's plan of care? Lidocaine IV infusion Transcutaneous pacing Trendelenburg position A 500-mL bolus of normal saline

Transcutaneous pacing The patient is in a third-degree atrioventricular (AV) block and requires some type of pacemaker. Lidocaine is contraindicated because it further decreases ventricular conduction. Placing the patient in a Trendelenburg position would increase the work of breathing and increase venous return, which could worsen the patient's condition. Administration of fluid boluses in heart failure patients would worsen the symptoms.

The nurse is placing electrocardiograph leads on a patient in a coronary care unit. Which are the correct lead positions? Select all that apply. V1 is placed on the fourth intercostal space on the right sternal border. V2 is placed on the fourth intercostal space on the left sternal border. V3 is situated halfway between V2 and V4. V4 is placed on the fifth intercostal space at the left midclavicular line. V5 is placed on the fifth intercostal space at the left midaxillary line. V5 is placed on the fifth intercostal space at the left anterior axillary line.

V1 is placed on the fourth intercostal space on the right sternal border. V2 is placed on the fourth intercostal space on the left sternal border. V3 is situated halfway between V2 and V4. V4 is placed on the fifth intercostal space at the left midclavicular line. V5 is placed on the fifth intercostal space at the left anterior axillary line. The V1 lead is placed on the fourth intercostal space at the right sternal border. The V2 lead is placed on the fourth intercostal space at the left sternal border. The V3 lead is placed halfway between V2 and V4. The V4 lead is placed on the fifth intercostal space at the left midclavicular line. The V5 lead is placed on the fifth intercostal space at the left anterior axillary line, not the midaxillary line.

A patient reports a fluttering feeling in the chest. The nurse assesses a rhythm of supraventricular tachycardia (PSVT), a heart rate of 150 beats per minute, and a BP of 120/60 mm Hg. Which treatments would the nurse expect to be added to the patient's plan of care? Select all that apply. Vagal stimulation IV β-blockers IV adenosine Emergent cardioversion IV calcium channel blockers

Vagal stimulation IV β-blockers IV adenosine IV calcium channel blockers Common vagal maneuvers include Valsalva, carotid massage, and coughing. Medications that may be used include β-blockers, adenosine, calcium channel blockers, and amiodarone. These drugs have impact on various phases of action potential. Adenosine decreases conduction through the atrioventricular (AV) nodes. β-blockers decrease automaticity of the sinoatrial (SA) node. If the patient becomes hemodynamically unstable and symptomatic, emergent cardioversion is considered.

The nurse provides education to a group of nursing students about cardiac conditions that are common causes of dysrhythmias. Which conditions would the nurse include in the teaching? Select all that apply. Valve disease Emotional crisis Conduction defects Accessory pathways Electrolyte imbalances

Valve disease Conduction defects Accessory pathways Dysrhythmia is a condition of abnormal heart rhythm caused by either abnormal conduction or abnormal formation of heart impulses. Several conditions are responsible for the development of dysrhythmia. The cardiac disorders that may lead to dysrhythmia involve valve disease, conduction defects, and accessory pathways. Emotional crisis and electrolyte imbalances are noncardiac conditions that may cause a dysrhythmia.

The nurse prepares to defibrillate a patient. Which dysrhythmia has the nurse observed in this patient? Ventricular fibrillation Third-degree AV block Uncontrolled atrial fibrillation Ventricular tachycardia with a pulse

Ventricular fibrillation Rationale: Defibrillation is always indicated in the treatment of ventricular fibrillation. Drug treatments are normally used in the treatment of uncontrolled atrial fibrillation and for ventricular tachycardia with a pulse (if the patient is stable). Otherwise, synchronized cardioversion is used (if the patient has a pulse). Pacemakers are the treatment of choice for third-degree heart block.

Which type of arrhythmia is associated with the absence of P waves on an electrocardiogram (ECG)? Sinus tachycardia Sinus bradycardia Ventricular fibrillation Type II second-degree atrioventricular (AV) block

Ventricular fibrillation Ventricular fibrillation is associated with the absence of P waves on an ECG. P waves are visible with sinus tachycardia, sinus bradycardia, and type II second-degree AV block rhythms.

Which medication is used to manage a complication of atrial fibrillation? Digoxin Warfarin Diltiazem Metoprolol

Warfarin A complication of atrial fibrillation is clot formation in the atria due to blood pooling. Warfarin is an anticoagulant to prevent this complication. Digoxin, diltiazem, and metoprolol are medications used to treat the actual abnormality/diagnosis of atrial fibrillation.

Which patient teaching points should the nurse include when providing discharge instructions to a patient with a new permanent pacemaker and the caregiver? (select all that apply) a. Avoid or limit air travel. b. Take and record a daily pulse rate. c. Obtain and wear a Medic Alert ID device at all times. d. Avoid lifting arm on the side of the pacemaker above shoulder. e. Do not use a microwave oven because it interferes with pacemaker function.

b, c, d Rationale: Pacemaker discharge teaching should include: Air travel is not restricted. The patient should tell airport security of the presence of a pacemaker because it may set off the metal detector. A hand-held screening wand should not pass directly over the pacemaker. Manufacturer information varies about the effect of metal detectors on pacemaker function. The patient should monitor the pulse and tell the HCP if it drops below a predetermined rate. The patient should have and wear a Medic Alert ID device at all times. The patient must avoid lifting the arm on the pacemaker side above the shoulder until approved by the HCP. Microwave ovens are safe to use. They do not interfere with pacemaker function.

A patient admitted with syncope has continuous ECG monitoring. An examination of the rhythm strip reveals the following: atrial rate 74 beats/min and regular; ventricular rate 62 beats/min and irregular; P wave normal shape; PR interval lengthens progressively until a P wave is not conducted; QRS normal shape. The priority nursing intervention would be to a. give epinephrine 1 mg IV push. b. prepare for synchronized cardioversion. c. observe for symptoms of hypotension or angina. d. apply transcutaneous pacemaker pads on the patient.

c Rationale: The rhythm is a second-degree atrioventricular (AV) block, type I (i.e., Mobitz I or Wenckebach heart block). It is characterized by a gradual lengthening of the PR interval. Type I AV block is usually a result of myocardial ischemia or infarction. It is typically transient and well tolerated. The nurse should assess for bradycardia, hypotension, and angina. The symptomatic patient may need atropine or a temporary pacemaker.

The ECG monitor of a patient in the cardiac care unit after an MI shows ventricular bigeminy with a rate of 50 beats/min. The nurse would a. perform defibrillation. b. administer IV amiodarone. c. prepare for temporary pacemaker insertion. d. assess the patient's response to the dysrhythmia.

d Rationale: A premature ventricular contraction (PVC) is a contraction originating in an ectopic focus in the ventricles. When every other beat is a PVC, the rhythm is called ventricular bigeminy. PVCs are usually a benign finding in patients with a normal heart. In patients with heart disease, PVCs may reduce the cardiac output and precipitate angina and heart failure, depending on the frequency. Because PVCs in coronary artery disease (CAD) or acute myocardial infarction indicate ventricular irritability, the patient's physiologic response to PVCs must be monitored. Assessing the patient's hemodynamic status is important for deciding the need for drug therapy.

A patient's heart monitor shows that every other beat is earlier than expected, has no visible P wave, and has a QRS complex that is wide and bizarre in shape. How will the nurse document the rhythm? a. Ventricular couplets b. Ventricular bigeminy c. Ventricular R-on-T phenomenon d. Multifocal premature ventricular contractions

ANS: B Ventricular bigeminy describes a rhythm in which every other QRS complex is wide and bizarre looking. Pairs of wide QRS complexes are described as ventricular couplets. There is no indication that the premature ventricular contractions are multifocal or that the R-on-T phenomenon is occurring.

The nurse provides education to a group of nursing students about nonemergent synchronized cardioversion. Which statement made by a student indicates effective learning? "The shock is delivered on the P wave." "Cardioversion is designed to stop the heart briefly." "Adenosine is given before the procedure for sedation." "After cardioversion, medications to regulate the heart rhythm are no longer needed."

"Cardioversion is designed to stop the heart briefly." Synchronized cardioversion is designed to send an electrical shock through the heart on the R wave. This stops the heart momentarily, allowing it to convert back to a normal sinus rhythm. In a nonemergent situation, sedation medicine is given before the procedure because of the pain of the electrical current passing through the chest wall. In nonemergent cases, sedation (e.g., IV midazolam) is given for the patient's comfort. Adenosine is an IV medication designed to treat paroxysmal supraventricular tachycardia (PSVT) by slowing the conduction through the atrioventricular (AV) node, allowing the heart to return to a normal rhythm, but it has no sedative effects. Postcardioversion, the patient will receive a prescription for cardiac medication to prevent recurrence of the tachyarrhythmia.

The registered nurse is reviewing a wireless electrocardiogram (ECG) monitoring system with a nursing student. Which statement made by the student indicates the need for further teaching? "It will not record the post-event portion of the ECG." "It can automatically save the pre-event portion of the ECG." "It continuously monitors and interprets the findings of patients." "It sends an alert when a patient's measurements fall outside the set parameters."

"It will not record the post-event portion of the ECG." The wireless ECG monitoring systems continue to record the post-event portion of ECG and send it to the health care provider. The wireless ECG monitoring systems can automatically save the pre-event portion of the ECG, continuously monitor and interpret the findings of a patient, and send an alert when patient rhythm or measurements fall outside the set parameters.

The nurse is analyzing patients' electrocardiogram (ECG) tracings. Which patient is at greatest risk for a stroke? REVIEW ECG STRIPS. 1 2 3 4

1 ECG tracing #1 shows an atrial flutter, which places the patient at risk for stroke. Thrombus formation because of stasis of ventricular blood is less likely in rhythms such as sinus rhythm with first-degree atrioventricular (AV) block (#2), junctional rhythm (#3), and paroxysmal supraventricular tachycardia (#4).

In which order do electrical impulses travel through the heart? 1.Sinoatrial node 2.Atrioventricular node 3.Bundle of His 4.Purkinje fibers 5.Internodal pathways

1. Sinoatrial node. 2. Intermodal pathways. 3. Atroventricular node. 4. Bundle of His 5. Purkinje fibers The conduction system of the heart consists of specialized neuromuscular tissue. The electrical impulse of the heart begins at the sinoatrial node in the upper right atrium. This impulse travels through the intermodal fibers and spreads over the atrial musculature. This causes atrial contraction. The impulse then reaches the atrioventricular (AV) node. From the AV node, the impulse moves down through the bundle of His and ends at the Purkinje fibers. Impulses from the Purkinje fibers cause ventricular contractions.

When computing a heart rate on a patient's electrocardiogram (ECG) tracing, the nurse counts 15 small blocks in an R-R interval. The rhythm is regular. Which heart rate in beats/minute would the nurse document? 60 75 100 150

100 Because each small block on the ECG paper represents 0.04 seconds, 1500 of these blocks represent one minute. By dividing the number of small blocks (15 in this case) into 1500, the nurse can calculate the heart rate in a patient whose rhythm is regular (in this case, 100). Sixty beats/minute, 75 beats/minute, and 150 beats/minute are incorrect answers.

What is the ventricular rate on the following electrocardiogram (ECG) tracing? Record the answer using a whole number rounded to the nearest 10 beats/min. ______beats/min

60 This ECG strip is marked in 1-second intervals. To calculate the ventricular rate, count the number of R-R intervals. On this rhythm strip, the R wave is the upright wave of the QRS complex. There are 6 R-R intervals, so the ventricular rate is 60 beats/min.

The nurse analyzes a patient's electrocardiogram (ECG) and determines that there are 8 R-R intervals in a span of six seconds. What should the nurse document as the patient's heart rate? Record the answer using a whole number. __ beats/min.

80 The heart rate can be calculated from an ECG by counting the number of R-R intervals in six seconds and multiplying that number by 10. In this case, the patient's ECG has eight R-R intervals. Therefore 8 multiplied by 10 is 80.

The nurse is monitoring the electrocardiograms of several patients on a cardiac telemetry unit. The patients are directly visible to the nurse, and all the patients are observed to be sitting up and talking with visitors. Which patient's rhythm would require the nurse to take immediate action? A 62-yr-old man with a fever and sinus tachycardia with a rate of 110 beats/min A 72-yr-old woman with atrial fibrillation with 60 to 80 QRS complexes per minute A 52-yr-old man with premature ventricular contractions (PVCs) at a rate of 12 per minute A 42-yr-old woman with first-degree AV block and sinus bradycardia at a rate of 56 beats/min

A 52-yr-old man with premature ventricular contractions (PVCs) at a rate of 12 per minute Rationale: Frequent premature ventricular contractions (PVCs) (>1 every 10 beats) may reduce the cardiac output and precipitate angina and heart failure, depending on their frequency. Because PVCs in CAD or acute myocardial infarction indicate ventricular irritability, the patient's physiologic response to PVCs must be monitored. Frequent PVCs may be treated with oxygen therapy, electrolyte replacement, or antidysrhythmic agents.

When preparing to defibrillate a patient, in which order will the nurse perform the following steps? a. Turn the defibrillator on. b. Deliver the electrical charge. c. Select the appropriate energy level. d. Place the hands-free, multifunction defibrillator pads on the patient's chest. e. Check the location of other staff and call out "all clear."

ANS: A, C, D, E, B This order will result in rapid defibrillation without endangering hospital staff.

When analyzing an electrocardiographic (ECG) rhythm strip of a patient with a regular heart rhythm, the nurse counts 30 small blocks from one R wave to the next. The nurse calculates the patient's heart rate as ____.

ANS: 50 There are 1500 small blocks in a minute, and the nurse will divide 1500 by 30.

A patient with dilated cardiomyopathy has new onset atrial fibrillation that has been unresponsive to drug therapy for several days. What topic should the nurse plan to include in patient teaching? a. Anticoagulant therapy b. Permanent pacemakers c. Emergency cardioversion d. IV adenosine (Adenocard)

ANS: A Atrial fibrillation therapy that has persisted for more than 48 hours requires anticoagulant treatment for 3 weeks before attempting cardioversion. This is done to prevent embolization of clots from the atria. Cardioversion may be done after several weeks of anticoagulation therapy. Adenosine is not used to treat atrial fibrillation. Pacemakers are routinely used for patients with bradydysrhythmias. Information does not indicate that the patient has a slow heart rate.

A patient develops symptomatic sinus tachycardia. Which drug will likely be included in the patient's treatment plan? Atropine Dopamine Adenosine Epinephrine

Adenosine Hypotension, dizziness, and dyspnea are symptoms of sinus tachycardia. Sinus tachycardia manifests as increased heart rate from 101 beats/minute to 180 beats/minute. Adenosine is used in the treatment of sinus tachycardia. Adenosine decreases the heart rate caused by inhibition of the vagus nerve and myocardial oxygen consumption. Anticholinergic drugs like atropine, dopamine, and epinephrine are the drugs of choice in the treatment of sinus bradycardia.

A patient reporting dizziness and shortness of breath is admitted with a dysrhythmia. Which medication, if ordered, requires the nurse to carefully monitor the patient for asystole? Digoxin Adenosine Metoprolol Atropine sulfate

Adenosine Rationale: IV adenosine is the first drug of choice to convert supraventricular tachycardia to a normal sinus rhythm. Adenosine is administered IV rapidly (over 1 or 2 seconds) followed by a rapid, normal saline flush. The nurse should monitor the patient's electrocardiogram continuously because a brief period of asystole after adenosine administration is common and expected. Atropine sulfate increases heart rate, while lanoxin and metoprolol slow the heart rate.

The nurse monitoring the electrocardiogram (ECG) of a patient with hyperthyroidism observes regular, sawtooth-shaped flutter waves with an atrial rate of 250 beats/minute. Which term would the nurse use to document this pattern? Atrial flutter Sinus bradycardia Sinus tachycardia Atrial fibrillation

Atrial flutter Atrial flutter is an atrial tachydysrhythmia identified by flutter (F) waves, a sawtooth-shaped pattern, with a 200 to 350 beats/minute atrial rate. In sinus bradycardia, the heart rate is less than 60 beats/minute, with regular rhythm and normal P waves. Sinus tachycardia is identified by 101 to 200 beats/minute, with regular rhythm and normal P waves. In atrial fibrillation, the atrial rate is 350 to 600 beats/minute, with irregular rhythm and fibrillatory (f) waves.

Which heart rate would the nurse expect in a patient with paroxysmal supraventricular tachycardia (PSVT)? Slower than 60 beats/min Between 61 and 100 beats/min Between 101 and 150 beats/min Between 151 and 220 beats/min

Between 151 and 220 beats/min PSVT is characterized by a heart rate of 151 to 220 beats/min. A heart rate of fewer than 60 beats/min is considered bradycardia. A rate of 100 beats/min is the upper limit for a normal heart rate, and a rate of 101 to 150 beats/min is the range for a sinus tachycardia.

Which conditions are indications for a patient to have a permanent pacemaker implanted? Select all that apply. Cardiomyopathy SA node dysfunction Coronary angioplasty Third-degree atrioventricular (AV) block New drug therapy that may cause bradycardia

Cardiomyopathy SA node dysfunction Third-degree atrioventricular (AV) block The permanent pacemaker is used in cardiomyopathy, SA node dysfunction, and third-degree AV block when the dysfunction of the electrical pathways is assumed to be permanent or irreversible. A temporary pacemaker may be required during coronary angioplasty and during drug therapy that may cause bradycardia.

While the nurse is administering furosemide via IV push (IVP), a patient becomes unresponsive. The patient's electrocardiogram (ECG) tracing shows the following. Which action would the nurse perform first? Cardiovert. Defibrillat. Check for a pulse. Administer oxygen.

Check for a pulse. The ECG tracing is ventricular tachycardia (VT). Ventricular tachycardia can either be with a pulse or pulseless. The treatment algorithm depends on whether the patient has a pulse. Therefore checking for a pulse is a priority. If the patient has a pulse, cardioversion and/or drug therapy is the priority. If the patient does not have a pulse, defibrillation is the priority. Oxygen may be administered, but it is not a priority.

A patient reports a new onset of jaw pain. The nurse obtains the following electrocardiogram (ECG) tracing. Which action should the nurse take? Administer prescribed hydrocodone. Contact the health care provider. Place the patient in the Trendelenburg position. Recognize the changes as indicating digitalis toxicity.

Contact the health care provider. The ECG tracing is showing ST elevation indicative of myocardial infarction (MI). The health care provider should be notified immediately so appropriate interventions can be prescribed. Morphine sulfate is the drug of choice for a patient experiencing an acute MI. Whenever possible, the patient experiencing an MI should be placed in a position promoting respirations. The Trendelenburg position inhibits respirations. Digitalis toxicity is characterized by ST segment depression, not elevation.

While ambulating, a patient's electrocardiogram (ECG) tracing changes from normal sinus rhythm with a heart rate of 90 beats/minute to the following tracing. Which action should the nurse take? Administer digoxin. Place the patient back into bed. Continue ambulating the patient. Notify the health care provider.

Continue ambulating the patient. It is a normal phenomenon for the heart rate to increase slightly during ambulation because of an increased demand for oxygen. Therefore the correct answer is to continue ambulating the patient. Digoxin does decrease the heart rate, but it is not given if the heart rate only increases with exertion. Placing the patient back into bed would not be appropriate in this situation. There is no need to notify the health care provider unless other symptoms occur.

A patient's electrocardiogram (ECG) has changed from a normal sinus rhythm to the following rhythm. The patient is sleeping, respirations are 16 breaths/min and unlabored, and the BP has dropped from 110/70 mm Hg to 104/68 mm Hg. Which action would the nurse take? Continue monitoring the patient. Perform an in-depth assessment. Notify the health care provider. Check the medical record for hyperkalemia.

Continue monitoring the patient. Sinus bradycardia can be a normal finding for athletes or patients when they sleep. Sinus bradycardia becomes clinically significant if the patient is symptomatic (hypotensive, chest pain, shortness of breath, change of level of consciousness). Because the respiratory status of the patient is stable and the BP is only slightly lower because the patient is sleeping, the nurse should continue monitoring the patient. Hyperkalemia is characterized by a peaked T wave, and in advanced stages a widened QRS complex, neither of which are demonstrated on this ECG tracing.

Which actions would esmolol have on a patient's cardiac conduction system? Select all that apply. Delays repolarization Accelerates repolarization Decreases automaticity of the sinoatrial (SA) node Slows the impulse conduction in the atrioventricular (AV) node Suppresses atrial dysrhythmias through an unknown mechanism

Decreases automaticity of the sinoatrial (SA) node Slows the impulse conduction in the atrioventricular (AV) node Esmolol is a β-adrenergic blocker that decreases the automaticity of the SA node and slows the impulse conduction in the AV node. Esmolol causes prolonged PR intervals. Potassium channel blockers delay repolarization. Class IB sodium channel blockers accelerate repolarization. Esmolol does not suppress atrial dysrhythmias through an unknown mechanism.

A patient is hospitalized with an acute myocardial infarction. The patient's cardiac rhythm suddenly changes from sinus tachycardia to the following rhythm. Which nursing action is priority? Defibrillate. Check for a pulse. Administer vasopressin. Perform synchronized cardioversion

Defibrillate. The electrocardiogram (ECG) tracing is ventricular fibrillation (VF), a lethal rhythm requiring a team of health care providers to provide interventions. VF results in an unresponsive, pulseless, and apneic patient. There should be no delay in starting compressions and using a defibrillator once available. It is not necessary for the nurse to check for a pulse; with VF, there will not be a pulse. Defibrillation and cardiopulmonary resuscitation (CPR) take priority over medications such as vasopressin. Synchronized cardioversion is a treatment for patients with a pulse; with VF, there will be no pulse.

The nurse is reviewing the use of defibrillation and synchronized cardioversion with students. Which information would the nurse provide? The patient is sedated before defibrillation is initiated. Defibrillation is the treatment of choice for ventricular fibrillation. Synchronized cardioversion is indicated to treat atrial bradydysrhythmias. Defibrillation is synchronized to deliver a shock during the QRS complex.

Defibrillation is the treatment of choice for ventricular fibrillation. Defibrillation is the treatment of choice for ventricular fibrillation and pulseless ventricular tachycardia. Patients are not sedated for defibrillation because a patient in ventricular tachycardia (VT) or pulseless VT will generally be unconscious. Synchronized cardioversion is the therapy of choice for the patient with hemodynamically unstable ventricular or supraventricular tachydysrhythmias. Defibrillation is not synchronized to deliver a shock during the QRS complex as there are not QRS waves visible in ventricular fibrillation.

A patient in asystole is likely to receive which drug treatment? Digoxin Lidocaine Epinephrine β-adrenergic blockers

Epinephrine Treatment of asystole consists of cardiopulmonary resuscitation (CPR) with initiation of advanced cardiac life support (ACLS) measures. These include definitive drug therapy with epinephrine and intubation. Digoxin is used for ventricular rate control. Lidocaine is used for premature ventricular contractions (PVCs). β-adrenergic blockers are used to slow the heart rate.

The nurse observes a flat line on the patient's monitor and the patient is unresponsive without pulse. What medications does the nurse prepare to administer? Lidocaine or amiodarone Digoxin and procainamide Epinephrine or vasopressin β-Adrenergic blockers and dopamine

Epinephrine or vasopressin Rationale: Normally, the patient in asystole cannot be successfully resuscitated. However, administration of epinephrine or vasopressin may prompt the return of depolarization and ventricular contraction. Lidocaine and amiodarone are used for ventricular tachycardia or ventricular fibrillation. Digoxin and procainamide are used for ventricular rate control. β-Adrenergic blockers are used to slow heart rate, and dopamine is used to increase heart rate.

Which cardiac problem is an indication for placement of a permanent pacemaker? Heart failure Edema after open-heart surgery Evaluation of tachydysrhythmias Acute inferior myocardial infarction

Heart failure A permanent pacemaker is indicated in a patient with heart failure. A temporary pacemaker may be indicated in patients recovering from open-heart surgery and inferior myocardial infarction, or in evaluation of tachydysrhythmias.

Which information in a patient's admission history may be the cause of the patient's sinus bradycardia? Select all that apply. Hypothermia Hyperglycemia Hyperthyroidism Calcium channel blockers Increased intracranial pressure

Hypothermia Calcium channel blockers Increased intracranial pressure The possible causes of sinus bradycardia include hypothermia, treatment with calcium channel blockers, and increased intracranial pressure. Hypothermia may cause reduced venous return, thereby causing bradycardia. Calcium channel blockers cause bradycardia by decreasing automaticity of the SA node and delaying the AV node conduction. The drug also reduces myocardial contractility. Increased intracranial pressure may suppress the cardiac centers in the brain, thus reducing the heart rate. Hyperglycemia causes tachycardia by causing systemic dehydration and acidosis. Hyperthyroidism increases the levels of thyroid hormone and increases the metabolism, resulting in tachycardia.

The nurse assesses an unresponsive patient and reviews the patient's ECG tracing. The nurse determines that the patient is experiencing pulseless electrical activity (PEA). In addition to identifying the cause, what is the priority nursing action? Defibrillate. Apply warm blankets. Assist with intubation. Initiate cardiopulmonary resuscitation (CPR)

Initiate cardiopulmonary resuscitation (CPR). PEA is a situation in which organized electrical activity is seen on the ECG, but there is no mechanical heart activity and the patient has no pulse. Treatment begins with CPR, followed by drug therapy and intubation. Correcting the underlying cause is critical to prognosis. Applying warm blankets would help with hypothermia but can be completed later. Intubation depends upon the patient's response to cardiopulmonary resuscitation. It is not appropriate to defibrillate; the activity on the ECG is organized.

A patient who is being tested for syncope has undergone the head-up tilt-test. After 30 minutes of testing, the patient's BP and heart rate (HR) did not respond, and no clinical symptoms were reproduced. Which medication will be given in a low-dose IV to provoke a response? Adenosine Midazolam Magnesium Isoproterenol

Isoproterenol A head-up tilt-test is a procedure used to determine the cause of fainting spells in a patient. In the head-up tilt-test, the patient is placed on a table supported by a belt across the torso and feet. The electrocardiogram and HR are recorded continuously, and BP is measured every three minutes throughout the test. If the patient's BP and HR responses are abnormal and faintness is reproduced, then the test is considered positive. If after 30 minutes there is no response, then the table is returned to the horizontal position and an IV infusion of low-dose isoproterenol is started to provoke a response. IV adenosine is the drug used in the treatment of paroxysmal supraventricular tachycardia. IV midazolam is used to sedate a patient before performing synchronized cardioversion. IV magnesium is used to treat polymorphic ventricular tachycardia with a prolonged baseline QT interval.

The nurse reviews the following electrocardiogram (ECG) tracing. Which condition is the likely cause of these findings? Medications Dehydration Fluid overload Myocardial ischemia

Myocardial ischemia Typical ECG changes that are seen in myocardial ischemia include ST segment depression and/or T wave inversion. Medications, fluid overload, and dehydration do not often affect the ST segment position on the ECG tracing.

The electrocardiogram (ECG) of a patient indicates P waves that are hidden in the preceding T waves and normal QRS complexes. Which condition is the patient experiencing? Ventricular fibrillation Junctional dysrhythmia Premature atrial contractions Premature ventricular contractions

Premature atrial contractions A premature atrial contraction occurs at the atrium and before the next sinus beat. A premature atrial contraction occurs in either the left atrium or right atrium and travels along the atria. The ECG of a premature atrial contraction usually shows hidden P waves in preceding T waves with prolonged PR interval. The QRS complex remains normal. The electrocardiogram of ventricular fibrillation has absent P waves and an undetectable PR interval and QRS complex. The ECG of junctional dysrhythmia shows a distorted P wave and reduced PR interval. The QRS complex remains normal. The ECG of premature ventricular contractions shows a rare occurrence of P waves. The PR interval cannot be measured with a disturbed and elongated QRS complex and T wave.

An experienced nurse teaches a group of new graduate nurses about temporary pacemakers. Which information would the nurse include about indications for a temporary pacemaker? Select all that apply. Heart failure Prophylaxis after open heart surgery Atrial fibrillation with slow ventricular response Acute anterior myocardial infarction (MI) with second- or third-degree atrioventricular (AV) block Acute inferior MI with symptomatic bradycardia and AV block

Prophylaxis after open heart surgery Acute anterior myocardial infarction (MI) with second- or third-degree atrioventricular (AV) block Acute inferior MI with symptomatic bradycardia and AV block A temporary pacemaker helps to maintain the normal pace of the heart when its electrical pathways are damaged. The power source of this device is placed outside the body. Temporary pacemakers are used after open heart surgery as prophylaxis. They are also indicated for use in patients with acute anterior MI with second- or third-degree heart block or bundle branch block or symptomatic bradycardia and AV block. Heart failure and atrial fibrillation are indications for a permanent pacemaker, not a temporary pacemaker.

The nurse observes ventricular tachycardia (VT) on the patient's monitor. What evaluation made by the nurse led to this interpretation? Unmeasurable rate and rhythm Rate 150 beats/min; inverted P wave Rate 200 beats/min; P wave not visible Rate 125 beats/min; normal QRS complex

Rate 200 beats/min; P wave not visible Rationale: VT is associated with a rate of 150 to 250 beats/min; the P wave is not normally visible. Rate and rhythm are not measurable in ventricular fibrillation. P wave inversion and a normal QRS complex are not associated with VT.

The nurse reviews a patient's electrocardiogram (ECG) tracing. Which interpretation would the nurse make? Sinus tachycardia Sinus bradycardia Premature atrial contraction Premature ventricular contraction

Sinus bradycardia Sinus bradycardia is a heart rhythm that originates from the sinus node. With this rhythm, the P wave precedes each QRS complex, and it has a normal shape and duration, normal PR interval, and QRS complex. The patient with sinus bradycardia generally has a heart rate of less than 60 beats/minute. Patients with sinus tachycardia have a heart rate in the range of 101 to 200 beats/minute. Patients with premature atrial contraction generally have an irregular rhythm and a different-shaped P wave. Patients with premature ventricular contraction have wide and distorted QRS complexes.

Assessment findings of a patient on a cardiac unit include decreased heart rate (HR), cardiac output (CO), and type I second-degree atrioventricular (AV) block. Which electrocardiogram (ECG) findings are characteristic of this rhythm? Select all that apply. Regular rhythm Slower ventricular rate Consistent R-R intervals Blocked QRS complexes Progressive lengthening of PR intervals

Slower ventricular rate Blocked QRS complexes Progressive lengthening of PR intervals Type I second-degree AV block is also called Mobitz I or Wenckebach phenomenon. It is depicted by the ECG as a progressive lengthening of the PR interval until there is a P wave without a QRS complex. Once a ventricular beat is blocked, the cycle repeats itself with progressive lengthening of the PR intervals until another QRS complex is blocked. The atrial rate remains normal, but ventricular rate may be slower, due to blocked QRS complex. This results in bradycardia. The cardiac rhythm is irregular. The R-R intervals are not consistent.

Which waveform in the electrocardiogram (ECG) tracing would be distorted when a patient has an electrolyte imbalance? P wave Q wave S wave T wave

T wave The T wave represents ventricular repolarization in an ECG. Disturbances (e.g., tall, peaked, inverted) in T waves can occur due to electrolyte imbalances, ischemia, and infarction. Disturbances in the P wave can be due to alterations in atrial conduction. Disturbances in the Q wave can occur due to myocardial infraction. Disturbances in the S wave do not affect the normal functioning of heart. Topics

A patient's electrocardiogram (ECG) tracing has changed from sinus tachycardia (ST) to the following rhythm. Which conclusion would the nurse make from the ECG finding? The patient is at risk for a pulmonary embolism. The patient is at risk for ventricular tachycardia. The patient is experiencing a myocardial infarction (MI). The patient is showing signs of an elevated potassium level.

The patient is experiencing a myocardial infarction (MI). ST elevation is a manifestation of an MI. ST elevation does not increase a patient's risk of a pulmonary embolism. A patient can go into ventricular tachycardia because of an MI, but it is not the main reason the nurse would notify the health care provider in this situation. Hyperkalemia is evidenced by a peaked T wave, not ST elevation.

The nurse obtains a 6-second rhythm strip and charts the following analysis: What is the correct interpretation of this rhythm strip? Sinus dysrhythmia Third-degree heart block Wenckebach phenomenon Premature ventricular contractions

Third-degree heart block Rationale: Third-degree heart block represents a loss of communication between the atrium and ventricles from atrioventricular node dissociation. This is depicted on the rhythm strip as no relationship between the P waves (representing atrial contraction) and QRS complexes (representing ventricular contraction). Whereas the atria are beating totally on their own at 70 beats/min, the ventricles are pacing themselves at 40 beats/min. Sinus dysrhythmia is seen with a slower heart rate with exhalation and an increased heart rate with inhalation. In Wenckebach heart block, there is a gradual lengthening of the PR interval until an atrial impulse is nonconducted and a QRS complex is blocked or missing. Premature ventricular contractions are the early occurrence of a wide, distorted QRS complex.

Which time period does the T wave in an electrocardiogram (ECG) represent? Ventricular repolarization Depolarization of both ventricles Passage of the electrical impulse through the atrium Time between ventricular depolarization and repolarization

Ventricular repolarization The ECG is commonly used to detect abnormal heart rhythms and to investigate the cause of chest pains. The T wave in the ECG should be upright; it represents time for ventricular repolarization. Time taken for depolarization of both ventricles is represented by QRS interval. Time for the passage of the electrical impulse through the atrium is represented by P wave. Time between ventricular depolarization and repolarization is represented by ST segment.


Kaugnay na mga set ng pag-aaral

Prep U chapter 40, Nursing Fluid & Electrolyte prep u, Chapter 4 - PrepU - Fluid and Electrolyte and Acid-Base Imbalances, Taylor's Chapter 39: Fluid, Electrolyte, and Acid-Base balance (PrepU), Prep-U Chapter 13: Fluid and Electrolytes: Balance and...

View Set

PHY 102-Chapters 8, 9, 11, & 12 Review

View Set

First Year, Second Semester mid term

View Set

F&B strategic planning and logistics ch 3 and 4

View Set

Principles of Real Estate 1 : Ch. 9

View Set

Chapter 22 Regulatory and Advisory Agencies

View Set

WGU D074 Principles of Accounting

View Set

Computer Systems Technology 1- Unit 1 (1.4-1.5)

View Set